SlideShare une entreprise Scribd logo
1  sur  42
Dermatologic and Cosmetic Surgery
1) The tensile strength of a wound 2 weeks after surgery compared to intact skin is:

A. 10%Correct Choice

B. 50%

C. 30%

D. 70%

E. 100%
The tensile strength of a wound approaches 10% of normal skin strength at 2 weeks
postoperatively. Wound healing is a process which takes many months, yet the tensile strength of a
wound never exceeds 70-80% of the tensile strength of intact skin. The use of subcuticular sutures
is therefore critical to minimize the tension on a healing wound and prevent scar widening


2) What risk does an organ transplant recipient have of developing melanoma?

A. Same risk as general population

B. Less risk than general population

C. 10- fold increased risk

D. 50-fold increased risk

E. 3-fold increased riskCorrect Choice
Organ transplant recipients are at risk for developing many cutaneous tumors. The risk of
melanoma is 3-fold that of the general population whereas these patients have a 65-fold increased
risk of developing squamous cell carcinoma


3) A Z- plasty is performed to:

A. Reorient a scar Correct Choice

B. Make use of skin from an area of laxity

C. Obtain wound eversion

D. Shorten the length of an excision

E. Close a wound on a convex surface
A Z-plasty is a type of transposition flap that may be used to reorient a scar, especially when the
scar crosses skin tension lines. It is also used to change the scar length or vector and ease scar
contractures


4) A Mohs surgery fellow experiences burning and tingling of her fingertips 6 months after starting
her fellowship. She most likely has been exposed to which one of the following chemicals?


A. Chlorhexidine gluconate

B. Povodine-iodine

C. Hematoxylin

D. Toluidine blue

E. Hexachlorophene Correct Choice




                                                  1
Hexachlorophene is an antibacterial agent that was first introduced in 1944. Its use was
discontinued in the 1970s when it was discovered to have neurotoxic side effects


5) The role of imiquimod in the treatment of superficial basal cell carcinoma is based on the
induction of which cytokine?


A. Tumor growth factor-alpha

B. Interferon-gamma Correct Choice

C. Tumor necrosis factor–beta

D. Interleukin-2

E. Interleukin-4
Imiquimod is an immunomodulator which induces the production of cytokines that are involved in
cell-mediated immunity. These cytokines include interferon-alpha, interferon-gamma, interleukin-1,
interleukin-10 and tumor necrosis factor-alpha. Induction of these cytokines results in antiviral and
antitumor activity in vivo


6) All of the following are branches of the internal carotid artery except:

A. Ophthalmic artery

B. Supratrochlear artery

C. Supraorbital artery

D. Angular arteryCorrect Choice

E. Dorsal nasal artery
Branches of the internal carotid artery include the opthamic artery, supraorbital artery, dorsal nasal
artery, supratrochlear artery, palpebral artery, and lacrimal artery. Branches of the external carotid
artery include the superficial temporal artery, facial artery (inferior labial, superior labial, angular
branches), maxillary artery, posterior auricular, and occipital artery. The external and internal
carotid arteries anastamose in the periorbital region


7) How long after cutaneous infiltration of lidocaine with epinephrine is maximal vasoconstriction
achieved?


A. 15 minutes Correct Choice

B. 7 minutes

C. 1 minute

D. 30 minutes

E. 1 hour
The use of epinephrine with local anesthesia has two main advantages. Firstly, the epinephrine is a
vasoconstrictor and assists in controlling bleeding during surgery. Secondly, and also a direct result
of its vasoconstrictive effects, epinephrine helps prolong the duration of the anesthetic agent 100%
to 200% by delaying its absorption from the surgical site. Although the anesthetic properties of
lidocaine take effect within the first few minutes of infiltration, the vasoconstrictive effects of
epinephrine require approximately 15 minutes to fully develop


8) Which of the following topical antibacterial agents demonstrates activity against Pseudomonas
species?




                                                   2
A. Erythromycin

B. Mupirocin

C. Polymyxin Correct Choice

D. Bacitracin

E. Clindamycin
Polymyxins are peptides produced by the organism Bacillus polymyxa. They are bactericidal against
Pseudomonas aeruginosa, Proteus mirabilis, Enterobacter, Klebsiella and Escherichia coli. Since they
provide little coverage against gram-positive organisms, polymyxins are often combined with other
antibacterial agents to increase their spectrum of activity


9) Healing by secondary intention will offer the most favorable cosmetic result at which location?

A. Malar cheek

B. Nasal tip

C. Chin

D. Medial canthusCorrect Choice

E. Forehead
Indications for healing of full-thickness skin wounds by secondary intention include infected wounds,
patients who refuse reconstructive surgery or who are poor surgical risks for reconstructive surgery,
and resection of tumors with high rates of recurrence. In addition, secondary intention, or
granulation, can be utilized when the cosmetic result is anticipated to be superior or equal to
reconstruction. Disadvantages of healing by granulation include prolonged wound healing time and
somewhat unpredictable cosmetic results. Anatomic sites lending themselves best to second
intention healing, with excellent results, are the concave areas on the face. These include the
medial canthus, the nose-cheek junction, the auricular concha, the preauricular cheek, and the
retroauricular scalp. The nasal tip, malar cheek, chin and forehead are all convex surfaces on the
face


10) Compared to intact skin, what is the tensile strength of a wound 6 months after surgery?

A. 30%

B. 90%

C. 50%

D. 70%Correct Choice

E. 100%
Tissue remodeling allows the host to develop a scar that is approximately 70% of the original
strength of the skin


11) The following flaps are types of transposition flaps except:

A. A to TCorrect Choice

B. Nasolabial

C. Bilobed

D. Z plasty

E. Rhombic flap




                                                  3
The Z plasty, Bilobed, Rhombic, nasolabial flaps are all examples of transposition flaps. In each of
these, the flap is transposed over a normal piece of skin to be placed in the recipient site. Whereas,
the A to T flap, is an advancement flap, in which tissue is moved in a linear direction to cover the
defect


12) Which of the following demonstrates the highest risk of metastasis?

A. SCC within a scar Correct Choice

B. SCC on the scalp

C. SCC on the nose

D. SCC on the ear

E. SCC on the lip
Location of tumor is an important risk factor for metastasis of squamous cell carcinoma. Compared
with a 10% likelihood of metastasis for tumors located on the ear or the lip, an SCC developing in
the scar, however, has been estimated to metastasize at a rate as high as 30-40%


13) A surgeon plans to repair a defect on the lower lip with a wedge resection. All of the following
statements are true regarding wedge resection of the lower lip except?


A. the vermillion border is marked prior to excision

B. the repair involves a two layered closure of skin and mucosaCorrect Choice

C. up to 1/3 of the lip can be excised and closed primarily

D. the mental crease should not be crossed

E. a V-shaped excision is often used
The lower lip has such great elasticity that full-thickness lesions that involve up to 1/3 of the lip can
be excised and repaired primarily with excellent cosmetic and functional results. This type of repair
requires a three layered closure of mucosa, skin and muscle.

Larrabee W. In: Principles of Facial Reconstruction. Lippincott-Raven: Philadelphia, 1995; 182-183


14) Endocarditis prophylaxis is recommended for each of the following except:

A. Previous bacterial endocarditis

B. Atrial septal defectsCorrect Choice

C. Mitral valve prolapse with regurgitation

D. Prosthetic valves

E. Hypertrophic cardiomyopathy
Preoperative antibiotics are recommended for endocarditits prophylaxis in those patients with
prostheitc valves, congenital cardiac malformations, previous bacterial endocarditis, rheumatic heart
disease with valve dysfunction, hypeertrophic cardiomyopathy, and mitral valve prolapse with
refurgitaion. Classically 1 gram of Dicloxacillin or cephalexin is given 1 hour preoperatively and an
additional 500 mg is given 6 hours post op. Clindamycin can be givenin those patients who are
penicillin allergic.

Preopertive antibiotics for endocarditis prohpylaxis are not required in those patients who have an
Atrial Septal Defect (ASD). pacemaker, rheumatic fever without valve dysfunction, or history of
Coronary Artery Bypass Graft (CABG




                                                    4
15) Which of the following is true regarding lidocaine:

A. Allergy most commonly occurs to propylene glycol preservatives

B. Maximum dose with no epinephrine is 7mg/kg

C. Beta blockers increase lidocaine levelsCorrect Choice

D. 1% lidocaine is equal to 1g/10ml

E. Duration with no epinephrine is 4-6 hours
Beta blockers increase lidocaine levels. The rest of the answers are false. 1% lidocaine is equal to
1g/100ml or 10mg/ml. Duruation of lidocaine with no epi is 30-60 minutes. Maximum dose of
lidocaine with no epi is 4.5 mg/kg, with epi it is 7 mg/kg. Allergy to lidocaine is most commonly due
to paraben preservatives


16) On average, how much does a full-thickness graft contract when removed from a donor site?

A. 30%

B. 15%Correct Choice

C. 80%

D. 50%

E. 60%
On the average, how much does a full-thickness graft may contract by about 15% when removed
from a donor site


17) The surgical instrument shown in the photograph is a:

A. Dermablade

B. No. 10 blade

C. No. 11 bladeCorrect Choice

D. No. 15 blade

E. Beaver blade
Surgical blades are made from either carbon steel or stainless steel and may be coated with Teflon
to reduce their drag through tissue. A no.11 blade, which is a narrow blade that ends in a sharp
point, is used to make stab incisions and is often employed in incision and drainage procedures


18) True statements regarding skin cancer in organ transplant recipients include all of the following
except:


A. Mohs micrographic surgery indicated for in-transit metastases Correct Choice

B. 65 fold increase in development of SCC compared with general population

C. Extent of tumor development related to degree of immunosuppression

D. Skin cancer is the most common cancer in transplant patients

E. Cutaneous malignancies develop 3-5 years after organ transplantation
In-transit metastases is a common manifestation of metastatic disease


19) Which of the following local anesthetics has the longest duration of action?


                                                  5
A. Mepivacaine

B. Benzocaine

C. Lidocaine

D. Procaine

E. BupivacaineCorrect Choice
Bupivacaine is an amide type of local anesthetic. Its duration of action is approximately 3-5 hours
and is the longest one listed. Following bupivacaine is mepivacaine, lidocaine, and procaine. The
duration of action is based on the amine portion of the molecule


20) Which needle shape is most commonly used in cutaneous surgery?

A. 1/4 circle

B. 5/8 circle

C. 3/8 circleCorrect Choice

D. compound curve

E. 1/2 circle
Needles are either straight or curved. Curved needles have their curvature described either as a
fraction of a circle or a compound curve. The greater the fraction of a circle, the more pronation and
supination of the wrist required by the surgeon to place the needle. The 3/8 circle needle is easy to
use in large, superficial areas and is the most commonly used needle for cutaneous surgery


21) Which of the following is the earliest symptom of lidocaine toxicity:

A. Perioral tinglingCorrect Choice

B. Nystagmus

C. Seizure

D. Tachycardia

E. Cyanosis
The maximum dosage of lidocaine is 4.5 mg/kg without epinephrine and 7.0 mg/kg with
epinephrine. Signs of lidocaine toxicity start at 1-5 microgm/ml with increased anxiety,
talkativeness, tinnitus, numbess/tingling around lips, metallic taste, double vision. Higher levels of
toxicity may cause nystagmus, muscle twitching, tremor and finally seizures and respiratory arrest


22) Which of the following cosmetic injectables does not illicit an inflammatory response?

A. Artecoll (polymethylmethacrylate)

B. Sculptra (poly L-lactic acid)

C. Radiance (hydroxyapatite)Correct Choice

D. Zyplast (collagen)

E. Silicone
Radiance is an injectable, biodegradable filler composed of calcium hydroxyapatite microspheres. It
is a normal constituent of bone and is therefore biocompatible. Current studies are examining the
role of hydroxyapatite in augmenting the craniofacial skeleton




                                                   6
23) Each of the following are branches of the internal carotid artery except:

A. Ophthalmic artery

B. Supraorbital artery

C. Supratrochlear artery

D. Angular artery Correct Choice

E. Dorsal nasal artery
The angular artery is a branch of the external carotid artery. It arises from the facial artery and
courses superiorly along the nasofacial angle until it reaches the area of the medial canthal tendon.
At this location, the angular artery anastamoses with the dorsal nasal branch of the ophthalmic
artery establishing a communication between the internal and external carotid arterial systems


24) Which of the following parameters determines the wavelength of a laser?

A. Fluence

B. Spot size

C. Lens length

D. Medium Correct Choice

E. Pulse duration
Laser light is monochromatic light that is emitted at a single wavelength. The wavelength of the
laser is determined by the medium in the optical cavity of the laser through which the light passes.
The medium may be solid, liquid or gas


25) The cutaneous lip and chin are divided into cosmetic units by which anatomic boundary?

A. Philtral ridge

B. Mental creaseCorrect Choice

C. Nasolabial fold

D. Vermillion border

E. Marionette lines
The term cosmetic unit is used to define regions of the face by specific characteristics such as color,
texture, presence of hair, and prominence of sebaceous glands. In dermatologic surgery, it is
preferable to conceal incisions within the boundary lines of cosmetic units. The mental crease, as
demonstrated in the photograph, divides the cutaneous lip and chin


26) Which statement is true regarding relaxed skin tension lines (RSTLs)?

A. incisional scars should be placed perpendicular to RSTLs

B. the long axis of a wound often lies in the direction of the RSTL Correct Choice

C. they lie parallel to the underlying muscles

D. they occur as a result of increased elastic tone

E. smiling minimizes the appearance of RSTLs
Relaxed skin tension lines are creases in the skin that are present at rest and develop as a
consequence of the decreased elasticity that occurs with aging and solar damage. These lines lie
perpendicular to the underlying musculature and can be accentuated by smiling or frowning. An



                                                      7
undermined circular wound will often form an oval shape due to muscular tension, and will have its
long axis oriented within these relaxed tension lines. Knowledge of RSTLs is important in cutaneous
surgery because placing incisions within these lines will ensure the most favorable cosmetic result


27) Pre-testing for allergy to collagen is required for which cosmetic filler?

A. Silicone

B. Artecoll Correct Choice

C. Radiance

D. Perlane

E. Cosmoderm
Artecoll is a nonbiodegradable injectable filler composed of microspheres of polymethylmethacrylate
which are suspended in bovine collagen. As with other types of injectable bovine collagen, patients
must be tested for allergy to bovine collagen prior to treatment


28) Damage to the zygomatic branch of the facial nerve leads to which of the following:

A. Facial asymmetry

B. Eyelid ectropion and inability to close eyelidCorrect Choice

C. Winged scapula

D. Inability to pucker lips

E. Unilateral eyelid ptosis
Damage to the zygomatic nerve causes eyelid ectropion and inability to close eyelid


29) A patient has a 2.0 cm surgical defect on the left lower eyelid after having Mohs surgery. Which
management option would be the least appropriate?


A. Primary closure

B. Secondary intentionCorrect Choice

C. Rhombic flap

D. Full-thickness skin graft

E. Tripier flap
In lower eyelid reconstruction, tension should be oriented parallel to the lower eyelid to prevent the
development of ectropion. Secondary intention would be the least appropriate management option
in this location because normal wound contraction would increase the risk of ectropion


30) Cosmetic units are defined by each of the following anatomic boundaries except:

A. Nasofacial sulcus

B. Glabellar region

C. Philtral ridge

D. Nasolabial folds

E. Nasal tip Correct Choice




                                                    8
The term cosmetic unit is used to define regions of the face by specific characteristics such as color,
texture, presence of hair, and prominence of sebaceous glands. In dermatologic surgery, it is
preferable to conceal incisions within the boundary lines of cosmetic units. In addition, moving skin
from one cosmetic unit to another should be avoided when possible to minimize the appearance of
apposed skin of dissimilar quality


31) The O to Z flap is what type of flap?

A. Pedicle

B. RotationCorrect Choice

C. Transposition

D. Bilobed

E. Advancement
The O to Z flap is a bilateral rotation flap in which tissue is moved from two ends to cover a central
defect. It is often used on the scalp or lower extremities in locations where tissue laxity is not
present.


32) A patient who had liposuction 5 weeks ago presents with multiple firm nodules at the cannula
insertion sites. You suspect the diagnosis is:


A. Herpes simplex infection

B. Cold panniculitis

C. Mycobacterial infectionCorrect Choice

D. Foreign body granuloma

E. Organized hematoma
Atypical mycobacterial infections are occurring with increasing frequency after cosmetic surgery
procedures. These infections typically occur 4-14 weeks after a procedure as a late-occurring
complication. Firm nodules at the treatment site or dehiscence of a previously healed wound may be
presenting signs of atypical mycobacterial infection


33) The main advantage of selecting 4-0 Vicryl rather than 5-0 Vicryl to suture a wound is:

A. Smaller suture diameter

B. Smaller needle

C. Increased tensile strength Correct Choice

D. Increased suture memory

E. Increased knot security
Sutures are classified according to the United States Pharmacopeia (USP) criteria. This classification
system specifies the diameter of a given suture material that is required to produce a certain tensile
strength. The smaller the cross-sectional diameter of a suture material, the higher the USP number
that is assigned. Thus, 4-0 Vicryl will have greater tensile strength and a larger cross-sectional
diameter than 5-0 Vicryl


34) All of the following statements are true regarding the ‘MCW Melanoma Cocktail’ except:

A. it demonstrates micrometastases in sentinel lymph nodes




                                                   9
B. it is performed intraoperatively

C. it is a mixture of polyclonal antibodies to tyrosinaseCorrect Choice

D. it is a mixture of monoclonal antibodies to MART-1

E. it is a mixture of monoclonal antibodies to Melan-A
The ‘MCW Melanoma Cocktail’ is an immunostain made up of monoclonal antibodies to MART-1,
Melan-A and tryrosine. It is used intraoperatively during sentinel lymph node biopsy and allows for
rapid and accurate determination of micrometastases.

Shidham VB et al. Optimization of an immunostaining protocol for the rapid intraoperative
evaluation of melanoma sentinel lymph node imprint smears with the ‘MCW Melanoma Cocktail’.
Cytojournal 2004;1(1):2


35) The “trapdoor effect” that can occur with a flap can be avoided by which of the following
techniques:


A. Using additional sutures

B. Avoid thinning the flap

C. Using a flap with length to width ratio of less than 3:1

D. Performing the flap in a two staged procedure

E. Wide underminingCorrect Choice
Wide undermining of the primary defect can release tension and prevent tenting or the trap door
deformity of the flap


36)The nasolabial two stage flap uses depends on which artery for flap survival:

A. Lateral Nasal artery

B. Angular arteryCorrect Choice

C. Superior labial artery

D. Inferior labial artery

E. Supratrochlear artery
Angular artery provides blood supply for the nasolabial transposition flap


37) A patient with end stage liver disease comes to your surgical practice. Which of the following
anesthetics should be used with this patient?


A. Mepivicaine

B. Lidocaine

C. BenzocaineCorrect Choice

D. Etidocaine

E. Prilocaine
Benzocaine, an ester, should be used as it is the only anesthetic mentioned above that is
metabolized by the kidney and not the liver




                                                   10
38) A 40 year-old woman from Southern California has wrinkles at rest on her forehead, scattered
telangiectasia on her nose and a few seborrheic keratoses on her chest. What category in Glogau's
photoaging classification scale does this patient represent?


A. Type 5

B. Type 2

C. Type 4

D. Type 1

E. Type 3 Correct Choice
Glogau’s photoaging scale is a classification system that employs clinical markers of photodamage
to determine an individual’s level of photoaging. The scale ranges from Type 1 through Type 4, with
Type 4 demonstrating the most extensive photodamage. Type 3 on the Glogau scale is classified as
advanced photoaging and describes a patient who is typically 50 years of age or older, has wrinkles
at rest, telangiectasias, obvious dyschromias and visible keratoses


39) Which post-operative complication is shown in the photograph?

A. Temporal nerve paralysis

B. Ectropion Correct Choice

C. Trap door deformity

D. Hypertrophic scar

E. Eclabium
Ectropion occurs when the eyelid margin turns outward away from the eye. In lower eyelid
reconstruction, tension should be oriented parallel to the lower eyelid to prevent the development of
ectropion


40) Which of the following facial rejuvenation techniques creates microthermal zones?

A. Radiofrequency

B. Intense pulsed light

C. Dermabrasion

D. Fractional photothermolysisCorrect Choice

E. CO2 laser
Fractional photothermolysis utilizes an infrared laser to create thousands of microthermal zones
with normal skin remaining between these zones. The zones of normal skin support rapid re-
epithelialization and result in much faster healing times than with traditional resurfacing procedures


41) Preliminary studies with phosphatidylcholine demonstrate efficacy in the treatment of:

A. Soft tissue augmentation

B. Acne scarring

C. Periorbital fat padsCorrect Choice

D. Postinflammatory hyperpigmentation

E. Superficial rhytids




                                                 11
Phosphatidylcholine is a lecithin-derived phospholipid which induces lipolysis when injected into
adipose tissue. Dissolution of fat is likely due to a detergent effect produced by the
phosphatidylcholine


42) A patient with a deep vertical frown line in the glabellar region requests Botox. Treatment will
mostly target which one of the following muscles?


A. Orbicularis oculi

B. Depressor superciliaris

C. Orbicularis oculi

D. Levator palpebrae superioris

E. Corrugator superciliaris Correct Choice
Although the procerus and orbicularis oculi muscles are contributory, the development of vertical
glabellar frown lines is mostly due to the overactivity of the corrugator supercilaris. These muscles
function to pull the eyebrow inferomedially


43) Botulinum toxin type A cleaves which protein in the presynaptic neuron?

A. Acetylcholine

B. Synaptobrevin (VAMP)

C. Snap-25Correct Choice

D. Serotonin

E. Syntaxin
Botulinum toxin type A is FDA approved for the treatment of glabellar wrinkles. Botulinum toxins act
by a three step process of binding, internalization by receptor medicated endocytosis and enzymatic
activation. It has specific light chain intracellular binding sites and different sites of action on
different SNARE (synaptosomal associated protein receptor [SNAP]) proteins. The SNARE proteins
are intimately involved in releasing acetylcholine at presynaptic terminals. Botulinum toxin A
cleaves SNAP-25 whereas B cleaves synaptobrevin


44) In a patient who is allergic to paraphenylinediamine, which of the following anesthetics should
be avoided?


A. BenzocaineCorrect Choice

B. Bupivicaine

C. Prilocaine

D. Mepivicaine

E. Etidocaine
Paraphenyelenediamine is a common allergen found in permanent hair dyes. In individuals who are
allergic to paraphenylendiamine, may also have allergic reactions to ester anesthetics, like
benzocaine. The other answer choices are amide anesthetics. Other crossreactants to
paraphenyelediamine include the preservative parabens, sulfonamides, and thiazide diuretics


45) The anatomic structure identified in the photograph is called the:

A. Philtral ridge




                                                  12
B. Nasal groove

C. Nasal ala

D. Soft triangle

E. ColumellaCorrect Choice
The columella is one of the cosmetic subunits of the nose. It is formed by the medial crura of the
alar cartilage


46) All of the following thrombotic complications have been reported after discontinuing aspirin
therapy prior to surgery except:


A. Stroke

B. Transient ischemic attack

C. Pulmonary embolismCorrect Choice

D. Cerebral embolism

E. Myocardial infarction
Kovich et al. reported thrombotic complications in patients who had stopped either coumadin or
aspirin peri-operatively. Thrombotic complications in patients who stopped warfarin included stroke,
TIA, myocardial infarction, cerebral embolism, death, DVT, pulmonary embolus, and blindness.
Thrombotic complications in patients who stopped aspirin included stroke, TIA, myocardial
infarction, cerebral embolism and death. No DVT or pulmonary embolus was reported


47) Which of the following immunosuppressive agents may exert a protective effect against the
development of skin cancers in organ transplant patients?


A. RapamycinCorrect Choice

B. Tacrolimus

C. Corticosteroids

D. Cyclosporine

E. Azathioprine
Rapamycin (also called sirolimus) is a macrolide antibiotic and a structural analog of FK 506. It is a
potent immunosuppressive agent which inhibits mTOR (a member of P13K family kinases). Despite
its immunosuppressive effects, preliminary data show a decreased incidence of skin cancer in organ
transplant patients treated with rapamycin and postulate that it may exert a protective effect
against cutaneous malignancies


48) Injection of Botox at the location identified in the photograph would denervate which muscle?

A. Frontalis

B. Corrugator superciliaris

C. Levator palpebrae superioris

D. Orbicularis oculi

E. ProcerusCorrect Choice
The development of vertical glabellar frown lines is due to the overactivity of the orbicularis oculi,
procerus and corrugator supercilaris muscles. These muscles function to pull the eyebrow
inferomedially




                                                   13
49) A safe dose of lidocaine with epinephrine when used in tumescent anesthesia is:

A. 3 mg/kg

B. 7 mg/kg

C. 4.5 mg/kg

D. 20 mg/kg

E. 50 mg/kg Correct Choice
The maximum recommended dosage of lidocaine in adults is 4.5 mg/kg without epinephrine, 7.0
mg/kg with epinephrine, and 55 mg/kg in tumescent anesthesia for liposuction


50) Which of the following determines the wavelength of a laser?

A. Spot size

B. Medium Correct Choice

C. Pulse duration

D. Fluence

E. Q switch
The wavelength of a laser is determined by the medium


51) Which of the following cosmetic injectables has the longest duration of action?

A. Sculptra (poly L-lactic acid)Correct Choice

B. Restylane (hyaluronic acid)

C. Cosmoderm (collagen)

D. Myobloc

E. Botox
Sculptra (called New-Fill outside of the US) is a biodegradable filler composed of poly-L-lactic acid,
the same material used in absorbable sutures. Preliminary studies have demonstrated longevity of
the filler at two years post-treatment.

Valantin MA et al. Polylactic acid implants (New-Fill) to correct facial lipoatrophy in HIV-infected
patients: results of the open-label study VEGA. AIDS. 2003;17:2471-2477


52) Which is the most important criterion for performing Mohs micrographic surgery?

A. Aggressive histology

B. Size of tumor > 2cm

C. Tumor recurrence

D. Tumor is contiguous Correct Choice

E. Location of tumor
Tumor that grows as a contiguous mass is an absolute requirement for Mohs micrographic surgery.
If a tumor exhibits discontiguous growth, discrete foci may be missed in the thin tissue layers and
thereby lead to a false-negative pathology interpretation. Size, location, recurrence and histology
are all indications for Mohs micrographic surgery rather than absolute requirements




                                                   14
53) The use of topical vitamin K has been shown to:

A. Induce keratinocyte differentiation

B. Increase collagen production

C. Minimize the appearance of telangiectasia

D. Reduce the severity of laser-induced purpura Correct Choice

E. Decrease the appearance of infraorbital pigmentation
Treatment of benign vascular lesions with the pulsed dye laser often produces significant
postoperative purpura. Topical vitamin K has been shown to decrease the severity of laser-induced
purpura although its mechanism of action has yet to be determined. No other cosmetic effects of
topical vitamin K have been proven to be statistically significant


54) Which of the following is the major antioxidant in the human epidermis?

A. Ascorbic acid

B. Coenzyme Q10

C. Alpha-tocopherol Correct Choice

D. Superoxide dismutase

E. Glutathione
Research into the preventive role of topical antioxidants in photoaging is based on the free radical
theory of aging. All of the choices above are antioxidants which are naturally found in the skin,
however, alpha-tocopherol is the major antioxidant in human epidermis. Epidermal depletion of this
vitamin has been shown to be an early and sensitive marker of environmental oxidative damage


55) The photograph demonstrates which of the following?

A. Composite graft

B. Full-thickness skin graft Correct Choice

C. Mohs layer

D. Split-thickness skin graft

E. Dog ear
A full-thickness skin graft (FTSG) is composed of the entire epidermis and dermis. Subcutaneous
tissue must be removed since the fat may compromise the viability of the graft


56) Treatment of acne scarring would be most effective with which modality?

A. Nd:YAG laserCorrect Choice

B. Intense pulsed light

C. Pulsed dye laser

D. Photodynamic therapy

E. Radiofrequency




                                                 15
Treatment with the nonablative 1064-nm Q-switched Nd:YAG provides significant improvement in
skin topography in patients with atrophic acne scars. The further improvements that are seen at 6-
month follow-up suggest that dermal remodeling is a process that continues long after treatment


57) Which structual component of local anesthetics (e.g. lidocaine) is responsible for the onset of
activity?


A. Amine end

B. None of the above

C. Length of the carbon chain

D. Aromatic ringCorrect Choice

E. Intermediate chain
Local anesthetics like lidocaine contain 3 principle structural elements. The aromatic ring determines
the onset of activity, the intermediate chain defines the class (amide vs. ester), and the amine end
is responsible for the duration of action


58) While removing sutures from an excision performed on the lateral neck, your patient reports
decreased sensation of the helix and lobule of the ipsilateral ear. Which of the following nerves was
likely injured during surgery?


A. Spinal accessory nerve

B. Greater auricular nerve Correct Choice

C. Auricular branch of vagus nerve

D. Lesser occipital nerve

E. Auriculotemporal nerve
Sensory innervation of the ear is provided by the greater auricular nerve, the auriculotemopral
nerve, the lesser occipital nerve and the auricular branch of the vagus nerve. The greater auricular
nerve is composed of the C2 and C3 branches of the cervical plexus and lies in the posterior triangle
of the neck. It provides sensation to the helix, antihelix, antitragus, posterior auricle and the lobule.
The spinal accessory nerve is a motor nerve which innervates the trapezius muscle. This nerve is
also subject to injury in the lateral neck but would cause motor deficits rather than sensory deficits


59) Paradoxical darkening has been associated with Q-switched ruby, Q-switched alexandrite, Q-
switched Nd:Yag treatment of which tattoo color?


A. Green

B. Black

C. Blue

D. Yellow

E. RedCorrect Choice
Paradoxic darkening of flesh-tone, red, and white tattoo inks with QSRL, Q-switched Nd:YAG, and
Q-switched alexandrite lasers has been reported. The tattoo pigments most associated with this
phenomenon are iron oxide and titanium dioxide. Ferric sulfide is converted to ferrous sulfate by the
laser


60) Which tattoo pigment is most commonly associated with allergic reactions, eczematous and
granulomatous?




                                                   16
A. Cadmium sulfide

B. Titanium dioxide

C. Cobalt

D. Mercuric sulfideCorrect Choice

E. Carbon
Allergic reactions have been reported with several different types of tattoo pigment. The most
commonly associated tattoo pigment however is mercuric sulfide. Tattoo with mercuric sulfide
produces a red color


61) Which dye is used to stain the Mohs section shown in the photograph?

A. Eosin

B. Toluidine blueCorrect Choice

C. Cytokeratin 7

D. Hematoxylin

E. Myeloperoxidase
Toluidine blue is the stain in the photographed Mohs section. Since mast cells have been associated
with tumor growth and are seen in increased numbers with basal cell carcinoma, staining of their
metachromatic granules with toluidine blue can be an effective way of visualizing basal cell
carcinoma on Mohs sections, particularly when the tumor is associated with fibrosis


62) The most common adverse reaction seen with betadine is:

A. Ototoxity

B. Seizures

C. Corneal damage

D. Allergic contact dermatitisCorrect Choice

E. Teratogenicity
Most common side effect seen with betadine is allergic contact dermatitis, secondary to the iodine
componen


63) Which tattoo pigment has been most commonly associated with pseudolymphomatous
reactions?


A. Mercuric sulfideCorrect Choice

B. Malachite

C. Cadmium sulfide

D. Phthalocyanine dyes

E. Cobalt
Some delayed type hypersensitivity reactions may lead to pseudolymphoma, most commonly
caused by red tattoo pigment (mercuric sulfide


64) Which nerve provides the sensory innervation to the tragus?



                                                17
A. AuriculotemporalCorrect Choice

B. Auricular branch of vagus

C. Facial

D. Glossopharyngeal

E. Greater auricular
The auriculotemporal nerve which is a branch of the mandibular branch of the temporal nerve
innervates the tragus. The sensory innervation to the auricle is provided by the greater auricular
nerve, and to a lesser extent the lesser occipital nerve. The external auditory canal, concha, and
posauricular sulcus supplied variably by 3 nerves - the auricular branch of the vagus (CNX), the
facial nerve (CN7), and the glossopharyngeal (CNIX


65) Treatment with which modality would be most effective for the patient shown?

A. Pulsed dye laser

B. Photodynamic therapy

C. Radiofrequency

D. Erbium:YAG laser

E. Intense pulsed lightCorrect Choice
The patient shown has multiple solar lentigines. Targeting the melanosomes in these benign,
superficial pigmented lesions would be most successfully accomplished with the intense pulsed light
source


66) The surgical instrument shown in the photograph is a:

A. Stevens tenotomy scissor Correct Choice

B. Metzenbaum scissor

C. Castroviejo scissor

D. Mayo scissor

E. Iris scissor
The instrument shown is a Stevens tenotomy scissor. This scissor is useful in dermatologic surgery
for tissue cutting and blunt dissection


67) All of the following are alpha-hydroxy acids except:

A. Tartaric acid

B. Citric acid

C. Glycolic acid

D. Lactic acid

E. Salicylic acidCorrect Choice
Alpha-hydroxy acids are naturally occurring carboxylic acids found in many foods. The alpha-
hydroxy acids include glycolic, lactic, malic, citric, and tartaric acids. Factors that determine the
intensity of the peel include the concentration of the acid, pH, degree of buffering, vehicle
formulation, frequency of application, conditions of delivery, and the length of time the acid is
placed on the skin. Salicylic acid is a type of beta-hydroxy acid




                                                   18
68) Formation of granulomas is a potential complication of treatment with:

A. Silicone Correct Choice

B. Hydroxychloroquine

C. Isotretinoin

D. Autologous fat

E. CO2 resurfacing
Silicone is a synthetic, viscous compound which is composed of long polymers of dimethylsiloxanes.
Silikon-1000 is one silicone product which is available in the United States and is FDA-approved for
the ophthalmic treatment of complicated retinal detachment. The number 1000 refers to the
product’s viscosity which is measured in centistokes. As a reference, water has a viscosity of 100
centistokes and mineral oil has a viscosity of about 350 centistokes. With respect to cosmetic
applications, silicone is not FDA-approved for soft tissue augmentation. In addition to the
occurrence of hypersensitivity reactions and product migration, there have been many reports of
granuloma formation after silicone injection, even many years post-treatment


69) Which ocular structure is at most risk of injury from an erbium:YAG laser?

A. Pupil

B. Iris

C. Retina

D. Lens

E. Cornea Correct Choice
The erbium:YAG laser is an ablative laser whose target chromophore is water. The cornea is an
aqueous structure and is therefore the most susceptible to damage from this laser


70) Activation of the procerus muscle causes

A. Accentuation of the melolabial folds

B. Perioral wrinkles

C. Periocular wrinkles

D. Wrinkles at the nasal rootCorrect Choice

E. Wrinkles on the forehead
The procerus muscle causes wrinkling at the nasal root and is often targeted with Botox therapy for
improved cosmesis


71) The cutaneous lip and chin are divided into cosmetic units by which anatomic boundary?

A. Vermillion border

B. Nasolabial fold

C. Mental creaseCorrect Choice

D. Philtral ridge

E. Marionette lines
The term cosmetic unit is used to define regions of the face by specific characteristics such as color,
texture, presence of hair, and prominence of sebaceous glands. In dermatologic surgery, it is



                                                  19
preferable to conceal incisions within the boundary lines of cosmetic units. The mental crease, as
demonstrated in the photograph, divides the cutaneous lip and chin


72) Treatment with which of the following lasers has been effective in psoriasis?

A. Pulsed dye laser

B. Alexandrite laser

C. Ruby Laser

D. Carbon dioxide laser

E. Excimer laswerCorrect Choice
The excimer laser is a 308 nm wavelength laser that has been used to treat psoriasis. The excimer
laser has some advantages over ultraviolet therapy. By treating only involved skin, higher doses can
be used and clearance may occur with fewer treatments


73) Which laser would not be a good choice for the treatment of a red cosmetic tattoo around the
lips?


A. Carbon dioxide

B. Pulsed dye

C. Nd:YAG

D. Q-switched rubyCorrect Choice

E. Alexandrite
Q-switched lasers can be effectively used to remove tattoos. However, immediate darkening of
white, flesh colored, and pink/red tattoos has been reported after treatment with Q-switched lasers.
Both ferric oxide and titanium dioxide (found in tattoo pigments) may be both be reduced after
heating by the laser and produce dramatic darkening


74) The anatomic structure identified in the photograph is called the:

A. Antihelix

B. Scaphoid fossa

C. Concha

D. Triangular fossaCorrect Choice

E. Tragus
The triangular fossa of the ear is shown in the photograph. This anatomic structure is bordered by
the crura of the antihelix


75) Which of the following cosmetic injectables can be seen on routine dental x-rays?

A. Botox

B. Radiance Correct Choice

C. Restylane

D. Zyplast

E. Cosmoderm




                                                 20
Radiance is an injectable, biodegradable filler which is composed of calcium hydroxylapatite
microspheres. Calcium hydroxylapatite is a normal constituent of bone and thus can be seen on
radiographic imaging. Botox is botulinum toxin; Zyplast is bovine collagen; Cosmoderm is a non-
animal form of collagen; and Restylane is a non-animal form of hyaluronic acid


76) The type of repair depicted in the photograph is a:

A. Full-thickness skin graft

B. Nasolabial flap

C. Rotation flap

D. Rhombic flap

E. Island pedicle flap Correct Choice
The island pedicle flap is a type of advancement flap commonly used for medial cheek defects. The
closure can often be camouflaged in the melolabial fold


77) The use of dermabrasion to improve the cosmesis of a scar is best performed how long after
the initial surgery?


A. 1 year

B. 1 week

C. 6 months

D. 6 weeksCorrect Choice

E. 3 weeks
Dermabrasion is the process of surgically planing or abrading the epidermis and dermis and is
usually carried out with a rapidly rotating wire brush or diamond fraise.
Following dermabrasion, reepithelialization from adnexal structures occurs with remodeling and
replacement of collagen bundles in the papillary and reticular dermis. This can result in
improvement of surgical or acne scars. When used in the post-operative period for surgical scars,
dermabrasion is usually carried out 6 to 8 weeks following the procedure. Regional dermabrasions
are routinely carried out under local anesthesia. Post-procedure complications include scarring,
pigmentary alterations, persistent erythema, and infection. Contraindications for dermabrasion
include recent use of isotretinoin and recent facelift, browlift or other procedure involving extensive
undermining. Reactivation of herpes labialis can be prevented in the lip area by administering
antiviral prophylaxis


78) Which cosmetic injectable provides dermal augmentation through the harvesting of a patient’s
fibroblasts?


A. Juvaderm

B. Dermalogen

C. Hylaform

D. IsolagenCorrect Choice

E. Restylane
Isolagen is an emerging technology whereby a patient’s own fibroblasts are isolated from a skin
biopsy, reproduced and then re-introduced into the patient’s treatment site


79) The tensile strength of a wound 6 months after surgery compared to intact skin is:



                                                  21
A. 70% Correct Choice

B. 15%

C. 30%

D. 50%

E. 100%
The tensile strength of a wound approaches 70% of normal skin strength at 8 weeks
postoperatively. Wound healing is a process which takes many months, yet the tensile strength of a
wound never exceeds 80% of the tensile strength of intact skin. It is therefore critical to use long-
lasting subcuticular sutures to minimize the tension on a healing wound and prevent scar widening


80) Epinephrine should be avoided with anesthetics in which of the following conditions?

A. Renal failure

B. Hypothyroidism

C. Liver failure

D. PheochromocytomaCorrect Choice

E. Diabetes Mellitus
Epinephrine should be avoided in patients with a pheochromocytoma


81) Which ocular structure is at most risk of injury from an erbium: YAG laser?

A. CorneaCorrect Choice

B. Iris

C. Retina

D. Vitreous humor

E. Lens
Injury from lasers may occur via direct or indirect ocular exposure. Damage is generally wavelength
specific. Laser that target hemoglobin or pigment may cause damage to the retinal pigment or
vasculature while lasers that target water as a chromophore (carbon dioxide and erbium) can
damage the cornea


82) Which one of the following lasers would be effective in the treatment of rhinophyma?

A. Carbon dioxide laserCorrect Choice

B. Alexandrite laser

C. KTP laser

D. Pulsed dye laser

E. Ruby laser
The carbon dioxide laser is a 10,600 nm laser that can be used to treat rhinophyma. The advantage
of the carbon dioxide laser over steel or dermabrasion is that the laser is relatively bloodless. The
erbium YAG can also be used to treat rhinophyma


83) A patient has a 1.5cm surgical defect on the right nasal sidewall that reveals the lateral
cartilage. Which management option would be the least appropriate?




                                                  22
A. Cheek transposition flap

B. Primary closure

C. Full-thickness skin graft Correct Choice

D. Forehead flap

E. Secondary intention
Full-thickness skin grafts (FTSGs) depend upon a viable, vascular bed at the recipient site in order
to survive. Exposed cartilage is an avascular tissue and therefore would not be able to support the
metabolic requirements of a FTSG


84) Aging skin demonstrates all of the following characteristic changes except:

A. Decreased number of Langerhans cells

B. Increase in subcutaneous fat Correct Choice

C. Hypertrophy of sebaceous glands

D. Decreased dermal collagen

E. Loss of elasticity
The loss and redistribution of subcutaneous fat is a characteristic finding of the aging face. The
forehead, temporal fossae, malar cheeks and perioral region are the most commonly affected areas.
Knowledge of senescent changes in fat distribution has altered the cosmetic surgeon’s approach of
the aging face to one which includes augmentation rather than one of simply lifting and tightening
the skin


85) The classic purpose for serrated scissors is:

A. For work on delicate areas around the eyes and the ears

B. For tissue undermining

C. For work on thicker tissues (back)

D. For sharp dissections of tissue

E. For gripping tissue while cuttingCorrect Choice
Serrated scissors are used primarily for gripping tissue to prevent sliding while cutting


86) Which of the following lasers may cause milia formation as a post-procedure complication?

A. Pulsed Dye

B. ErbiumCorrect Choice

C. ND-Yag

D. KTP

E. Excimer
The erbium (2940 nm) and carbon dioxide (10,600 nm) lasers may both cause milia formation after
laser skin resurfacing


87) Which of the following surgical prepatory solutions is teratogenic?

A. Povidine-iodine



                                                     23
B. Ethyl and Isopropyl alcohol

C. Benzalkonium chloride

D. Chlorhexidine

E. HexachloropheneCorrect Choice
Hexachlorophene or pHisoHex has shown to be toxic to developing embryos in animal studies and
thus is prohibited for use in pregnant females. Chlorhexidine is a common ingredient in oral
rinses(Peridex) and surgical prep solutions(Hibiclens) can cause ototoxicity if in contact with the
middle ear and ocular toxicity if in contact with the eyes. Ethyl alcohol is effective against both gram
positive and gram negative organisms, but only when the skin is dry. Both Povidine-Iodine
(Betadine)and Benzalkonium chloride (Zephiran) can cause an allergic contact dermatitis


88) What is the most appropriate management of the following surgical complication?

A. Antibiotic therapy

B. Observation Correct Choice

C. Incision and drainage

D. Debridement

E. Pulse dye laser
If the epidermal surface of a graft becomes black and necrotic, it does not necessarily signify graft
failure. The epidermal portion of the graft may slough with subsequent re-epithelialization. In this
situation, the best treatment is observation


89) A physician using a carbon dioxide laser to treat verruca should be aware that all of the
following complications can occur except:


A. Recurrence of lesion

B. Ocular damage

C. Transmission of viral disease

D. PurpuraCorrect Choice

E. Scarring
The carbon dioxide laser uses a 10,600 nm wavelength to target water as a chromophore. Lasers
that target water may damage the cornea. The carbon dioxide laser can be used to destroy
epidermal and dermal lesions such as warts. HPV virus has been recovered in the laser plume after
treatment with carbon dioxide laser. Bovine papilloma virus recovered in the plume was even found
to transmit disease to calf skin. Scarring and recurrence have also been found to be potential
complications


90) A Z-plasty is performed to:

A. Correct dog-ears

B. Obtain wound eversion

C. Shorten the length of an excision

D. Decrease tension on a wound Correct Choice

E. Make use of excess tissue




                                                  24
A Z-plasty is a transposition flap used most commonly in the treatment of contractures and scars.
The main advantages to performing a Z-plasty are to decrease the tension on a wound, change the
orientation of a scar, and camouflage a scar by breaking it up into smaller components


91) Which of the following cosmetic injectables is FDA-approved for the treatment of lipoatrophy?

A. Botox

B. Radiance

C. Silicone

D. SculptraCorrect Choice

E. Isolagen
Sculptra (called New-Fill outside of the US) is a biodegradable filler composed of poly-L-lactic acid,
the same material used in absorbable sutures. It is biocompatible and nonallergenic and was
approved by the FDA in August 2004 for the treatment of HIV-associated lipoatrophy


92) A history of anaphylaxis is a contraindication to treatment with which cosmetic injectable?

A. Cosmoderm

B. Radiance

C. Sculptra

D. RestylaneCorrect Choice

E. Myobloc
Restylane is a hyaluronic acid gel produced by the Streptococcus species of bacteria. The package
insert for Restylane warns that its use is contraindicated in patients with “severe allergies
manifested by a history of anaphylaxis or the presence of multiple severe allergies


93) What part of the eye may be damaged by exposure to irradiation from the carbon dioxide laser?

A. Sclera

B. Lens

C. Retina

D. CorneaCorrect Choice

E. Iris
The carbon dioxide laser operates at a wavelength of 10,600 nm and targets water as a
chromophore. Because of the high water content of the cornea, it may be damaged by exposure to
irradiation from the carbon dioxide laser. Exposure to the erbium may cause corneal damage as well


94) All of the following statements are true regarding nerves blocks on the face except:

A. Anesthesia is placed around a nerve trunk

B. Lidocaine with epinephrine may be used

C. Numbness occurs in areas other than the operative site

D. Hemostasis is achieved using epinephrineCorrect Choice

E. Tissue distortion is minimized




                                                  25
While regional nerve blocks offer many advantages in cutaneous surgery, hemostasis is not one of
them. Since the anesthetic is injected distant to the operative site, the vasoconstrictive effects of
the epinephrine, and hence hemostasis, are not provided at the surgical site


95) The post-operative complication shown in the photograph is most commonly seen with which
type of reconstruction?


A. Full-thickness skin graft

B. Rhombic flap

C. Nasolabial flap Correct Choice

D. Secondary intention

E. Rotation flap
Trap door deformity is believed to result from insufficient undermining. This surgical complication is
most often associated with the nasolabial transposition flap. Intralesional corticosteroids may be
beneficial in improving the cosmetic outcome


96) The pain associated with Botulinum A Exotoxin injection is attributed to the:

A. Needle gauge

B. Preservative-free salineCorrect Choice

C. Temperature

D. PH

E. Exotoxin
Botulinum A exotoxin is used for multiple reasons, most often for the treatment of dynamic facial
lines. In a double-blind, randomized controlled study, investigators found that botulinum A exotoxin
reconstituted with preservative-containing saline less painful than with preservative-free saline


97) An M-plasty is performed to:

A. Make use of excess tissue

B. Reorient a scar

C. Decrease tension on a wound

D. Obtain wound eversion

E. Shorten the length of an excision Correct Choice
An M-plasty is a variation of the fusiform excision where either one or both ends of the ellipse are
modified. The main advantages to performing an M-plasty are to shorten the length of an excision
and to correct dog-ears


98) Botulinum toxin A (Botox) is FDA-approved for the treatment of:

A. Platysmal bands

B. Migraine headaches

C. HyperhidrosisCorrect Choice

D. Masseter hypertrophy




                                                  26
E. Crow’s feet
Botox has been recently FDA-approved for the treatment of hyperhidrosis.
A CPT procedure code has been assigned to treatment with Botox for this indication with variable
reimbursement rates by insurance companies


99) The antiptosis subdermal suspension threads used in facial rejuvenation are composed of:

A. Polyglycolic acid

B. Poly-L-lactic acid

C. Polydioxanone

D. Polyglactin

E. PolypropyleneCorrect Choice
The Aptos subdermal suspension thread technique is a minimally invasive procedure which targets
the ptotic changes seen with facial aging. 2-0 and 3-0 polypropylene threads are tunneled in the
dermis to lift and suspend the skin and subcutaneous tissue


100) Which one of the following best estimates the percent strength of a wound one month after
surgery with a primary closure?


A. 90%

B. 70%

C. 80%

D. 30%

E. 50%Correct Choice
One month after surgery with a primary closure a wound is approximately at 50% strength. Two
weeks after surgery it is at 5% strength. Three weeks after surgery it is at 20%. It never reaches
100% of its original strength, its maximum strength is 90%


101) Topical vitamin K has been shown to:

A. Increases epidermal differentiation

B. Improve skin hydration

C. Decrease epidermal pigmentation

D. Reduce severity of postoperative purpuraCorrect Choice

E. Improve fine winkling
Topical vitamin K reduced post later treatment purpura when applied after (not before) treatment of
telangectasia with the 585 nm pulsed dye laser


102) Which type of collagen is the first to be deposited in a healing wound?

A. Type I collagen

B. Type II collagen

C. Type III collagenCorrect Choice

D. Type IV collagen




                                                 27
E. Type VII collagen
Coagulation/Inflammation, tissue formation, and scar/remodeling are the three phases important in
wound healing. Collagen deposition occurs during the second phase. Fibroblasts migrate into the
wound along the fibrin-fibronectin matrix deposited in the initial clot. The fibroblasts subsequently
produce type I and III collagen, elastin, and proteoglycans. Collagen type III is the predominant
collagen in early wound healing


103) All of the following statements are true regarding imiquimod (Aldara) except:

A. It is FDA-approved for the treatment of actinic keratoses

B. It is FDA-approved for the treatment of a 1.9 cm superficial basal cell on the chest

C. It is FDA-approved for the treatment of a 2.0 cm superficial basal cell on the back

D. It is FDA-approved for the treatment of a 1.5 cm superficial basal cell on the leg

E. It is FDA-approved for the treatment of a 1.2 cm superficial basal cell on the scalpCorrect Choice
Imiquimod (Aldara) is FDA-approved for the treatment of actinic keratoses and the treatment of
superficial basal carcinoma. With respect to basal cell carcinoma, treatment is indicated for primary
tumors that are 2.0 cm or less, and that are located on areas of the body excluding the face, scalp
and anogenital region


104) A patient has a 1.8 cm nodular basal cell carcinoma on his neck. Which of the following
criteria would be an indication for Mohs micrographic surgery?


A. Anatomic location

B. Size of tumor > 1.5 cm

C. Tumor is contiguous

D. Previous radiationCorrect Choice

E. Histologic subtype
Previous irradiation is an indication for Mohs surgery because the effects of radiation are similar
histologically and biologically to scarring. Specifically, the tissue can show fibrosis and alter the
tumor’s growth pattern. In terms of location, the neck is not a high risk site. With respect to
pathology, nodular basal cell is not an aggressive subtype of basal cell carcinoma. Size of tumor > 2
cm would be an indication for Mohs. Finally, that the tumor grows contiguously is not an indication
but an absolute requirement for Mohs surgery


105) Which porphyrin is targeted in aminolevulinic acid-photodynamic therapy?

A. Uroporphyrinogen

B. Coproporphyrinogen

C. Porphobilinogen

D. Protoporphyrin IXCorrect Choice

E. Coproporphyrin III
Protoporphyrin IX is the substrate for the final rate-limiting step of heme synthesis. This enzymatic
reaction is catalyzed by ferrochelatase. The preferential uptake of aminolevulinic acid by cancerous
and pre-cancerous cells results in higher concentrations of protoporphyrin IX. This can then be
selectively targeted by photodynamic therapy


106) Which of the following sun protection agents is currently seeking FDA approval?


                                                  28
A. MexorylCorrect Choice

B. Titanium dioxide

C. Avobenzone

D. Zinc oxide

E. Octocrylene
Mexoryl is considered the best UVA sunscreen currently available. Its broad-spectrum
characteristics allow sunscreens to be manufactured with very high SPF factors. Various
formulations of mexoryl have long-been available in Europe, Canada, Australia, Asia, Mexico and
South America


107) Which of the following local anesthetic agents should not be used in children?

A. Benzocaine

B. PrilocaineCorrect Choice

C. Etidocaine

D. Mepivicaine

E. Bupivicaine
Prilocaine should bot be used in children given the risk of
methemoglobinemia.Methemoglobulinemia in children can occur from exposure to oxidizing
substances such as aniline dyes, prilocaine or pyrimidine. In methemoglobinemia, the iron in
hemoglobin is oxidized from the ferrous state (Fe 2+) to ferric (Fe3+) resulting in the inability to
transport oxygen and carbon dioxide. Clinically, this condition results in cyanosis


108) When can a patient who undergoes follicular-unit hair transplantation expect hair growth of
the grafted follicles to begin?


A. 2 to 3 months Correct Choice

B. 7 to 10 days

C. 6 months

D. 9 months

E. 1 month
The use of follicular-unit grafts, which contain one to four hair follicles, represents the advancement
in both surgical technique and aesthetic outcome in the field of hair transplantation. The grafted
hair follicles typically begin to grow within 8 to 10 weeks of implantation and are expected to
survive for the individual’s lifetime


109) Which of the following is a potential side effect of ambulatory tumescent liposuction?

A. Pulmonary embolus

B. Abdominal perforation

C. Decreased appetite

D. Muscle atrophy

E. Breast enlargement Correct Choice
Breast enlargement is a relatively common and unexpected side effect of tumescent liposuction.
Since the majority of these patients report increased breast size in the absence of weight gain,




                                                  29
some authors postulate shifting hormone ratios as the etiology of this paradoxical breast
augmentation. Abdominal perforation, respiratory failure and pulmonary embolus are complications
that are seen almost exclusively in liposuction patients that receive general anesthesia or
intravenous sedation


110) A patient with a squamous cell carcinoma of the helical rim had excisional surgery followed by
closure of the wound with an advancement flap. Which would be the most appropriate antibiotic for
this patient to receive postoperatively?


A. Trimethoprim-sulfamethoxazole

B. Cephalexin

C. Ciprofloxacin Correct Choice

D. Erythromycin

E. Dicloxacillin
Infection after cutaneous surgery is typically due to Staphylococcal and Streptococcal organisms.
However, Pseudomonas aeruginosa may be normal flora of the ear and thus can potentially
complicate ear surgery by causing severe infection of the external ear canal (malignant otitis
externa). Ciprofloxacin would be the antibiotic of choice for this patient because it is effective
against Pseudomonas species


111) The repair demonstrated in the photograph is a:

A. Advancement flap Correct Choice

B. Rotation flap

C. Full-thickness skin graft

D. Split-thickness skin graft

E. Transposition flap
The postauricular flap is useful for repairing extensive defects of the helical rim, particularly when
cartilage is involved. It is an example of an advancement flap


112) The use of EMLA cream is contraindicated in patients with which of the following?

A. Sickle cell anemia

B. Peripheral neuropathy

C. MethemoglobinemiaCorrect Choice

D. Atopic dermatitis

E. Deomycin allergy
The most serious adverse effect of eutectic lidocaine and prilocaine (EMLA) is methemaglobinemia.
A metabolite of prilocaine oxidizes hemoglobin to methemoglobin, which is less efficient in release of
oxygen leading to tissue hypoxia. Patients with congenital or idiopathic methemoglobinemia or
infants under 1 year of age are at higher risk for the development of this side effect


113) The type of repair depicted in the photograph is a:

A. Rotation flap

B. Island pedicle flap




                                                   30
C. Full-thickness skin graft

D. Rhombic flap Correct Choice

E. Nasolabial flap
The rhombic flap is a type of transposition flap. The classic rhombic flap is designed with two 60
degree angles and two 120 degree angles. The point of maximum tension is at the closure of the
donor site


114) In organ transplant recipients, all of the following factors increase susceptibility to the
development of skin cancer except:


A. CD8 lymphocytopeniaCorrect Choice

B. Fair skin (Fitzpatrick types I-III)

C. Duration of immunosuppression

D. History of chronic sun exposure

E. History of HPV infection
All organ transplant recipients are at increased risk for the development of cutaneous malignancies.
However, the above-mentioned factors, with the exception of choice E, place these individuals at
further risk. CD4 lymphocytopenia, rather than CD8, is another identified risk factor as is older age,
history of actinic keratoses and history of skin cancer


115) The anatomic structure identified in the photograph is called the:

A. Philtral ridgeCorrect Choice

B. Cupid’s bow

C. Vermillion border

D. Soft triangle

E. Columella
The philtral ridges are located at the upper middle lip and form part of the aesthetically important
cupid’s bow


116) What is the tattoo pigment responsible for most lichenoid reactions?

A. Titanium dioxide

B. Mercuric sulfideCorrect Choice

C. Carbon

D. Chromates

E. Iron oxide
Lichenoid tattoo reactions are not as common as eczematous reactions, both of which are most
commonly caused by mercuric sulfide. Lichenoid reaction are likely to be mediated by delayed
hypersensitivity to a lymphocytic T-cell infiltrate


117) Which part of the eye is most likely to be damaged to exposure to a pulsed dye laser?

A. Sclera




                                                   31
B. Cornea

C. RetinaCorrect Choice

D. Lens

E. Conjuctiva
The pulsed dye laser operates at 585-nm and targets hemoglobin as a chromophore. It can pass
through the cornea and damage choroidal and retinal vasculature. Several infrared pigment-specific
lasers (eg, ruby, alexandrite, Nd:YAG) may also damage the retina by targeting the retinal pigment


118) All of the following are true statements regarding the immunohistochemical stain Melan-A
except:


A. Reliably stains desmoplastic melanoma Correct Choice

B. Sensitive marker for melanocytic nevi

C. Antigen present in 80-100% of melanomas

D. Effective in frozen and paraffin-embedded tissue

E. Recognized by CD8+ T cells
E Melan-A is a 22-kDa cytoplasmic melanosome-associated glycoprotein also known as MART-1
(melanoma antigen recognized by T cells). It is a sensitive marker for both melanocytic nevi and
melanoma and may be used on frozen tissue sections as well as paraffin-embedded tissue. Although
the antigen is present in > 80% of melanomas, it does not reliably stain desmoplastic or spindle cell
melanomas


119) A patient with a basal cell carcinoma on the lower eyelid has Mohs surgery. Once tumor-free
margins were obtained, the surgeon repairs the wound with a full thickness skin graft. Which of the
following statements is true about this repair method?


A. Healing by secondary intention would minimize the risk of ectropion

B. Graft should be at least 25% smaller than the defect

C. Graft should be the same size as the defect

D. Graft rarely survives in this location

E. Graft should be at least 25% larger than the defectCorrect Choice
To account for wound contraction and minimize the risk of ectropion, full thickness skin grafts on
the lower eyelid should be sized such that the graft is at least 25% larger than the actual size of the
wound.

Jewett BS and Shockley WW. Reconstructive options for periocular defects. Otolaryngol Clin North
Am 2001; 34(3):601-625


120) Which tattoo pigment has most commonly been associated with phototoxic reactions?

A. Titanium dioxide

B. Mercuric sulfide

C. Cadmium sulfideCorrect Choice

D. Carbon

E. Iron oxide




                                                  32
Cadmium sulfide produces a yellow tattoo. Phototoxic reactions are most commonly caused by
cadmium sulfide tattoo pigment. Red tattoos have also been associated with phototoxic tattoo
reactions. However, this is thought thought to be due to the addition of cadmium sulfide which
enhances and brightens the red color


121) Which of the following lasers has the greatest depth of penetration in the skin?

A. Diode (800 nm)

B. CO2 (10,600 nm)

C. Erbium: YAG (2940 nm)

D. Pulsed dye laser (585 nm)

E. Nd: YAG (1064 nm) Correct Choice
The Nd: YAG laser emits energy at 1064nm near the infrared range and penetrates the skin to the
level of the deep dermal blood vessels. The target chromophore for this laser is melanin, thus its
primary use is in treating lesions such as nevus of Ota and removing black tattoo pigment. Although
both the erbium:YAG and CO2 lasers have longer wavelengths than the Nd: YAG, they are ablative
lasers which only penetrate to the level of the stratum corneum and superficial epidermis,
respectively


122) Which of the following suture materials is most likely to cause infection?

A. PDS (polydioxanone)

B. Silk Correct Choice

C. Prolene (polypropylene)

D. Nylon

E. Vicryl (polyglactin)
Silk is a nonabsorbable braided suture which has been shown to aid in the production of infection. It
is believed that the interstices of braided or twisted suture materials enhance the potential for
developing infection by providing privileged sites which harbor bacteria


123) A patient with a large nasal tip defect had reconstruction of his wound with a forehead flap.
After what postoperative interval should the patient schedule the inset of this flap?


A. 2 months

B. 6 months

C. 3 weeks Correct Choice

D. 3 months

E. 1 week
The forehead flap is often the reconstructive option of choice for large nasal defects. The flap has a
vertically-oriented blood supply which is supplied primarily from the supratrochlear arteries. Pedicle
division and inset of this flap are typically performed 3 weeks after the initial surgery


124) Which of the following criteria carries the worst prognosis for a patient with a squamous cell
carcinoma?


A. Size of tumor > 1 cm




                                                  33
B. Perineural invasion

C. Depth of invasion > 4 mm Correct Choice

D. Immunosuppression

E. Anatomic location
Many factors contribute to the aggressive behavior of cutaneous squamous cell carcinoma. For
example, tumors in immunosuppressed patients and tumors with the histologic subtypes
desmoplastic and acantholytic are often biologically aggressive squamous cell carcinomas.
Compared with other risk factors, depth of invasion >4 mm and size > 2cm demonstrate the
greatest risk for metastasis.


125) The relaxed skin tension lines overlying the scapula are:

A. Oriented horizontally

B. Parallel to the underlying muscle groups

C. Oriented verticallyCorrect Choice

D. Do not exist over the scapula

E. Oriented circumferentially
Relaxed skin tension lines (RSTLs) are creases in the skin that are present at rest and develop as a
consequence of the decreased elasticity that occurs with aging and solar damage. These lines lie
perpendicular to the underlying musculature. They tend to run vertically on the upper back whereas
on the central trunk, they are typically circumferential. Knowledge of RSTLs is important in
cutaneous surgery because placing incisions within these lines will ensure the most favorable
cosmetic result


126) Which of the following cosmetic injectables binds water to create volume, has a low allergic
reaction profile, and lasts 6 to 12 months?


A. Silicone

B. Calcium hydroxylapatite

C. Human-derived collagen

D. Botulinum toxin

E. Hyaluronic acid Correct Choice
Hyaluronic acid is a natural component of human connective tissue. As a cosmetic filler, hyaluronic
acid is an attractive alternative to currently available filler substances: its ability to bind water
creates volume and plumps the skin; its duration of action of approximately 6-12 months is
appreciably longer than conventional collagen; and since hyaluronic acid is chemically identical
across all species, hypersensitivity reactions are rare


127) Which of the forms of electrosurgery can work in a wet field?

A. ElectrocauteryCorrect Choice

B. Electrodessication

C. Electrocoagulation

D. Electrosection

E. Electrofulguration
Electrocautery is the only one to work in a wet field as it has no current and works via heat



                                                  34
128) Which of the following complications has been reported with infraorbital injections of Botox?

A. FestooningCorrect Choice

B. Photophobia

C. Blindness

D. Astigmatism

E. Nystagmus
Festooning of the lower eyelid is a recently reported complication of infraorbital Botox injections.
The affected patient had a prior lower lid blepharoplasty which likely weakened the orbicularis oculi
muscle. The author postulates that festooning resulted from further muscle weakness due to Botox
treatment


129) When is it appropriate to use this surgical technique?

A. To shorten the length of the scar

B. To change direction of a scar that is deformingCorrect Choice

C. When there is insufficient tissue for rotation flap

D. To prevent bleeding

E. When it is desirable not to extend an ellipse
Z-plasty is used to change the direction of scar that is deforming or to elongate a scar that is
contricting. Common sites are medial canthus, side of the face and forehead


130) Which of the following sutures is the first to be absorbed?

A. Polyglycolic acid

B. Polydioxanone

C. CatgutCorrect Choice

D. Polypropylene

E. Polyglactin 910
Catgut has a variable rate of absorption but typically lasts about 7-14 days. Thus, it would be the
first suture to be absorbed. Polyglactin 910 (Vicryl) and polyglycolic acid (Dexon) are both absorbed
in about 90 days. Polydioxanone (PDS) is an absorbable monofilament which lasts approximately
180 days. Polypropylene (Prolene) is not an absorbable suture.



131) The most appropriate suture for closing a wound on the buccal mucosa is:

A. Braided nylon

B. SilkCorrect Choice

C. Prolene

D. PDS

E. Monocryl




                                                    35
The most appropriate suture for closing a wound on the buccal mucosa is silk. Other sutures may be
used as well, but classically, silk is used because of its soft nature


132) The most appropriate laser to treat a port wine stain on an infant’s cheek would have which of
the following wavelengths?


A. 488 nm

B. 585 nm Correct Choice

C. 532 nm

D. 810 nm

E. 694 nm
The 585nm pulsed dye laser (PDL) targets intravascular oxyhemoglobin and is considered the
treatment of choice for most benign vascular lesions. The original PDL had a wavelength of 577 nm
which was later modified to 585 nm to achieve deeper penetration yet still maintainvascular
specificity


133) Which of the following absorbable sutures lasts the longest?

A. Polydioxanone Correct Choice

B. Catgut

C. Polypropylene

D. Polyglycolic acid

E. Polyglactin 910
Polydioxanone (PDS) is an absorbable monofilament which lasts approximately 180 days. Catgut
has a variable rate of absorption but typically lasts about 7-14 days. Polyglactin 910 (Vicryl) and
polyglycolic acid (Dexon) are both absorbed in about 90 days. Polypropylene (Prolene) is not an
absorbable suture


134) The most important function of a postoperative wound dressing is:

A. Provide a barrier to infection

B. Decrease tension on the wound

C. Create a moist environment

D. Prevent suture dehiscence

E. Immobilize the wound Correct Choice
Providing hemostasis to a wound is the most important function of a wound dressing. This is
particularly critical in the first few hours postoperatively when the vasoconstrictive effects of
epinephrine are wearing off and the wound is at risk of bleeding or hematoma formation. The
provision of a barrier to infection and creation of a moist, occlusive environment are also necessary
functions of a dressing but are not as critical as immobilizing the wound and ensuring hemostasis


135) While contemplating the repair of a large cheek defect after Mohs surgery, you begin to
anesthetize an elderly lady who weighs 110 lbs. What is the maximum amount of 1% lidocaine with
2.5% epinephrine that this patient can receive?


A. 500 ml




                                                  36
B. 350 ml

C. 35 ml Correct Choice

D. 50 ml

E. 10 ml
he maximum recommended dosage of 1% lidocaine with 2.5% epinephrine in adults is 7mg/kg. The
patient weighs 110 lbs (or 50 kg) which allows her 350 mg or 35 ml (1% lidocaine has 10mg of
lidocaine perml) of the anesthetic


136) Which potential complications is seen exclusively with phenol peels?

A. Laryngeal edema Correct Choice

B. Prolonged erythema

C. Milia

D. Toxic shock syndrome

E. Hypopigmentation
The principle concept of chemical peeling is to utilize a chemical agent to produce a controlled
wound in the skin. Many different types of side effects can occur with chemical resurfacing and are
typically related to the depth of the wound. Complications which are common to all peeling agents
include: milia; acne; pigmentary changes; contact dermatitis; scarring; infection; prolonged
erythema; textural changes and cold sensitivity. Unlike other peeling agents, phenol has the
potential to cause severe adverse reactions which are exclusive to its use only. They include
atrophy, cardiac arrhythmias and laryngeal edema


137) Reticulate eythema is a side effect seen with which treatment?

A. Cryotherapy

B. Pulsed dye laser

C. Diode laserCorrect Choice

D. Mesotherapy

E. Sclerotherapy
Reticulate erythema is a recently-reported side effect of diode laser treatment. High energy fluences
and a history of chilblains are believed to be predisposing risk factors. Laser treatment should be
discontinued at the first sign of this complication


138) Alopecia neoplastica is most commonly associated with which malignancy?

A. Lymphoma

B. Squamous cell carcinoma

C. Breast carcinoma Correct Choice

D. Colon carcinoma

E. Melanoma
While various malignancies have a predilection for metastasis to the scalp where they can cause
hair loss, alopecia neoplastica is most commonly associated with metastatic breast carcinoma.


139) The ‘snap test’ is performed prior to surgery at which anatomic location?


                                                 37
A. Mouth

B. Neck

C. Ear

D. Hand

E. EyelidCorrect Choice
The snap test determines the laxity of the lower eyelid. It should be performed prior to surgery,
laser resurfacing or Botox injections of the lower eyelid to assess the potential for ectropion
formation.
Ratner D et al. Cutaneous laser resurfacing. J Am Acad Dermatol 1999; 41(3):365-389
140) Which anesthetic has the shortest duration of action?

A. Tetracaine

B. ProcaineCorrect Choice

C. Mepivacaine

D. Bupivacaine

E. Prilocaine
Procaine has the shortest duration of action, only lasting about 30-60 minutes. It is an ester
anesthetic and is commonly used in dentistry
141) For patients with AJCC stage III and IV melanoma, which of the following adjuvant treatments
has shown the greatest increase in survival?


A. Melacine

B. CanvaxinCorrect Choice

C. Interferon alpha-2B

D. Dacarbazine

E. M-Vax
The current paradigm for treatment of advanced-stage melanoma includes tumor reduction via
surgery followed by adjuvant systemic therapy to manage residual occult disease. Although
interferon alpha 2B and dacarbazine are FDA-approved adjuvant therapies, neither has significantly
increased survival rates. Canvaxin is a polyvalent vaccine that contains more than 20 tumor and
melanoma-associated antigens. Currently in phase 3 trials, postsurgical immunotherapy with
Canvaxin has shown in retrospective matched-pair studies to prolong overall survival of stage III
and IV melanoma patients by more than 175 percent


142) What is the ratio of sodium bicarbonate to 1% lidocaine in a buffered lidocaine solution?

A. 1:25

B. 1:5

C. 1:10Correct Choice

D. 1:1

E. 1:2
The use of sodium bicarbonate in 1% lidocaine reduces the acidity of the local anesthetic.
Unbuffered lidocaine has pH of approximately 5.5 -7.0 and lidocaine with epinephrine has a pH of
approximately 3.5 – 5.0. The addition of sodium bicarbonate at a ratio 1:10 raises the pH to a more
physiologic level, thereby reducing the stinging sensation associated with injection. However, the
addition of bicarbonate also reduces the shelf-life of the lidocaine



                                                 38
143) The use of imiquimod (Aldara) for the treatment of superficial basal cell carcinoma is
advocated at which treatment regimen?


A. Five times per week for 4 weeks

B. Five times a week for 6 weeks Correct Choice

C. Three times a week for 10 weeks

D. Three times per week for 4 weeks

E. Three times a week for 6 weeks
Imiquimod (Aldara) was FDA-approved in 2004 for the treatment of superficial basal cell carcinoma.
The recommended treatment schedule is once daily, five times per week for a total of six weeks.
Geisse J et al. Imiquimod 5% cream for the treatment of superficial basal cell carcinoma: results
from two phase III, randomized, vehicle-controlled studies
144) When would use of this technique be most beneficial?

A. To shorten the length of the woundCorrect Choice

B. To prevent post-operative bleeding

C. To prevent pincushioning

D. To prevent post-operative infection

E. To increase the width of the wound
M-plasty is used to shorten wound length and is often when it is not desirable to extend the ellipse
into a nearby structure


145) Which anatomic structure is likely to be severed during this repair?

A. Marginal mandibular nerve

B. Labial artery Correct Choice

C. Mental nerve

D. Angular artery

E. Buccal nerve
The photograph demonstrates a wedge resection which is used to repair defects of the lower lip.
During this repair, the labial artery will be severed since it courses superficially between the mucosa
and the underlying muscle


146) The rhytids identified in the photograph are referred to as:

A. Marionette lines

B. Bunny linesCorrect Choice

C. Nasolabial folds

D. Crow’s feet

E. Relaxed skin tension lines
Bunny lines are located on the dorsum of the nose and are formed by the contraction of the nasalis
muscle


147) Which of the following topical antibacterial agents may cause neutropenia?


                                                  39
A. Silver sulfadiazine Correct Choice

B. Mupirocin

C. Bacitracin

D. Polymyxin

E. Povodine-iodine
Silver sulfadiazine is most commonly used to prevent infection in second and third-degree burn
patients. Rare cases of leukopenia, neutropenia and kernicterus have been reported in patients
using this topical antibacterial agent. Consequently, silver sulfadiazine should be used cautiously
and avoided in infants, nursing mothers and pregnant women. Silver sulfadiazine also has the
potential to cause a hypersensitivity reaction in patients with sulfa allergy


148) Which material is used to coat the jaws of the needle holder in the photograph?

A. Tungsten carbideCorrect Choice

B. Copper

C. Gold

D. Nickel

E. Stainless steel
Tungsten carbide is believed to be a harder more durable material that allows for better grasping of
the needle. Needle holders with tungsten carbide jaws usually have gold-plated handles


149) Jessner’s solution contains all of the following except:

A. Ethanol

B. Salicylic acid

C. Resorcinol

D. Lactic acid

E. Glycolic acidCorrect Choice
Jessner’s solution is a combination of resorcinol, salicylic acid, and lactic acid in ethanol that is used
as a superficial peeling agent. The advantage of Jessner’s solution is that timing is unnecessary and
neutralization is not performed. The lactic acid is an alpha hydroxy acid


150) The following are true statements regarding the design of an excisional ellipse except:

A. Ellipse includes pathologic and normal tissue

B. Incision lines are placed in relaxed skin tension lines

C. Long axis of ellipse is oriented perpendicular to free margins

D. Length of ellipse is at least 3 times its total width

E. The angle at each of the apices is 15ºCorrect Choice
Thoughtful design of an elliptical excision is important for obtaining optimum tissue diagnosis, tumor
clearance, wound closure and cosmetic result. The above statements are all true except that the
angles at the apices of the ellipse should be 30º
151) Blue nodules in the skin are a potential complication of which cosmetic injectable?




                                                    40
A. Silicone

B. Radiance (hydroxyapatite)

C. Cosmoderm (collagen)

D. Restylane (hyaluronic acid)Correct Choice

E. Sculptra (poly L-lactic acid)
Although hyaluronic acid is believed to be inert in humans, rare reports exist of granulomatous
foreign body reactions to this material. Skin biopsy of a Restylane nodule demonstrates
multinucleated giant cells surrounding a blue amorphous material
152) Which of the following is true regarding a Baker-Gordon peel?

A. Croton oil is considered the most important component for efficacyCorrect Choice

B. It contains resorcinol

C. 88% phenol is the most important component for efficacy

D. It contains salicytic acid

E. It is considered a medium depth peel
Croton oil is considered the most important component for efficacy. It is a epidermolytic that
enhances penetration. A Baker Gordon peel is a deep peel that contains 88% phenol, tap water,
croton oil, and septisol


153) The surgical instrument shown in the photograph is a:

A. Towel clamp

B. Iris scissor

C. HemostatCorrect Choice

D. Gradle scissor

E. Needle holder
A hemostat is shown in the photograph. This instrument is commonly used in cutaneous surgery for
clamping off blood vessels


154) The use of EMLA cream is contraindicated in patients with which of the following?

A. Neomycin allergy

B. Methemoglobinemia Correct Choice

C. Atopic dermatitis

D. Peripheral neuropathy

E. Sickle cell anemia
EMLA is a topical anesthetic composed of a eutectic mixture of 2.5% lidocaine and 2.5% prilocaine.
The major concern when using EMLA is the potential risk of methemoglobinemia. Patients with
glucose-6-phosphate deficiency and patients taking methemoglobin-inducing agents (dapsone,
phenytoin, sulfonamides) are more susceptible to developing methemoglobinemia. In addition, care
should be taken when using EMLA in infants less than 3 months of age because of the incomplete
maturation of their NADH-methemoglobinemia reductase system
155) Signs and symptoms of lidocaine toxicity include all of the following except:




                                                 41
A. OtotoxicityCorrect Choice

B. Circumoral numbness

C. Seizure

D. Slurred speech

E. Nystagmus
The first signs of lidocaine toxicity are CNS symtpoms that resemble inebriation with alcohol. These
symptmoms include stupor, dysarthria, circumoral numbenss and dizziness. Further increases in
toxicity leads to nausea, metallic taste, twitching, and seizures. Ototoxicity is not one of the toxic
manifestations seen with lidocaine toxicity. Without epinephrine the maximum dose of lidocaine
4mg/kg. For a 70 kg individual, this is 300 mg or 30 ml of a 1% lidocaine solution. For a
preparation of lidocaine without epinephrine the maximum dose is 7mg/kg. For a 70 kg individual,
this is 500 mg or 50 ml of a 1% solution
156) To avoid injury to the facial nerve, undermining in the region of the temple should be
performed at which anatomic level?


A. Reticular dermis

B. Subcutaneous fat Correct Choice

C. Above the frontalis muscle

D. Above superficial fascia (SMAS)

E. Dermal-epidermal junction
In the region of the temple, lateral to the eyebrow, the temporal branch of the facial nerve lies in
the superficial fascia. Once above the eyebrow, the nerve is located deep to the frontalis muscle. In
the region of the temple, it is therefore prudent to undermine in the superficial subcutaneous fat
157) Which of the following histochemical stains can be used intraoperatively during Mohs
micrographic surgery to confirm the diagnosis of extramammary Paget's disease?


A. S-100

B. Myeloperoxidase

C. Toluidine blue

D. Neuron-specific enolase

E. Cytokeratin 7 Correct Choice
Cytokeratin 7 is a structural component of the cytoskeleton that stains poorly differentiated tumors
of the epithelium. This low molecular weight cytokeratin positively stains Paget cells. Neuron-
specific enolase stains merkel cell carcinoma, toluidine blue stains mast cells and myeloperoxidase
stains cells with myeloid differentiation such as leukemia cells. S-100 protein is a non-specific stain
that is commonly used as an adjunctive marker in the diagnosis of melanoma




                                                  42

Contenu connexe

Tendances (20)

Ganyang MCQ Dermatology
Ganyang MCQ DermatologyGanyang MCQ Dermatology
Ganyang MCQ Dermatology
 
Introduction To Dermatopathology
Introduction To DermatopathologyIntroduction To Dermatopathology
Introduction To Dermatopathology
 
Acne presentation
Acne presentationAcne presentation
Acne presentation
 
Skin Aging
Skin AgingSkin Aging
Skin Aging
 
Aging face ppt
Aging face pptAging face ppt
Aging face ppt
 
Acne, rosacea and hidradenitis suppurativa
Acne, rosacea and hidradenitis suppurativaAcne, rosacea and hidradenitis suppurativa
Acne, rosacea and hidradenitis suppurativa
 
ETAS_MCQ_09 pediatric dermatology
ETAS_MCQ_09 pediatric dermatologyETAS_MCQ_09 pediatric dermatology
ETAS_MCQ_09 pediatric dermatology
 
Acne Vulgaris
Acne VulgarisAcne Vulgaris
Acne Vulgaris
 
Dermatological emergency
Dermatological emergencyDermatological emergency
Dermatological emergency
 
Acne scar treatment
Acne scar treatmentAcne scar treatment
Acne scar treatment
 
Skin fungal infection
Skin fungal infectionSkin fungal infection
Skin fungal infection
 
Melanogenesis.doc 2
Melanogenesis.doc 2Melanogenesis.doc 2
Melanogenesis.doc 2
 
Seminar principles of topical therapy
Seminar principles of topical therapySeminar principles of topical therapy
Seminar principles of topical therapy
 
Dermatology board review
Dermatology board reviewDermatology board review
Dermatology board review
 
Psoriasis part2
Psoriasis part2Psoriasis part2
Psoriasis part2
 
Dermatosis bacterianas
Dermatosis bacterianasDermatosis bacterianas
Dermatosis bacterianas
 
11. emergency dermatology
11. emergency dermatology11. emergency dermatology
11. emergency dermatology
 
Retinoids in dermatology seminar
Retinoids in dermatology seminarRetinoids in dermatology seminar
Retinoids in dermatology seminar
 
Epidermal kinetics
Epidermal kineticsEpidermal kinetics
Epidermal kinetics
 
Cutaneous mucinoses
Cutaneous mucinosesCutaneous mucinoses
Cutaneous mucinoses
 

En vedette

ETAS_MCQ_05 dermatopathology
ETAS_MCQ_05 dermatopathologyETAS_MCQ_05 dermatopathology
ETAS_MCQ_05 dermatopathologyDerma202
 
ETAS_MCQ_11 disorder of hair and nails
ETAS_MCQ_11 disorder of hair and nailsETAS_MCQ_11 disorder of hair and nails
ETAS_MCQ_11 disorder of hair and nailsDerma202
 
ETAS_MCQ_10 manifestations of systemic diseases1
ETAS_MCQ_10 manifestations of systemic diseases1ETAS_MCQ_10 manifestations of systemic diseases1
ETAS_MCQ_10 manifestations of systemic diseases1Derma202
 
ETAS_MCQ_03 a genodermatoses
ETAS_MCQ_03 a genodermatosesETAS_MCQ_03 a genodermatoses
ETAS_MCQ_03 a genodermatosesDerma202
 
Teks ucapan menteri pelajaran malaysia perutusan 2011
Teks ucapan menteri pelajaran malaysia perutusan 2011Teks ucapan menteri pelajaran malaysia perutusan 2011
Teks ucapan menteri pelajaran malaysia perutusan 2011Azman Adnan
 
Teks ucapan hari guru 2013
Teks ucapan hari guru 2013Teks ucapan hari guru 2013
Teks ucapan hari guru 2013Chon Seong Hoo
 
Dermatology without pics
Dermatology without picsDermatology without pics
Dermatology without picsess_online
 
Teks ucapan wakil murid sempena hari guru
Teks ucapan wakil murid sempena hari guruTeks ucapan wakil murid sempena hari guru
Teks ucapan wakil murid sempena hari guruAzmi Bokhari
 
Contoh Farewell Speech (ucapan/pidato perpisahan)
Contoh Farewell Speech (ucapan/pidato perpisahan)Contoh Farewell Speech (ucapan/pidato perpisahan)
Contoh Farewell Speech (ucapan/pidato perpisahan)Aufiya Tsalitsa
 
41499971 teks-ucapan-majlis-persaraan-guru-besar
41499971 teks-ucapan-majlis-persaraan-guru-besar41499971 teks-ucapan-majlis-persaraan-guru-besar
41499971 teks-ucapan-majlis-persaraan-guru-besarbagero
 
SELAMAT BERSARA
SELAMAT BERSARASELAMAT BERSARA
SELAMAT BERSARAImsamad
 
Phototherapy treatment protocol
Phototherapy treatment protocolPhototherapy treatment protocol
Phototherapy treatment protocolDerma202
 
ucapan-perpisahan
 ucapan-perpisahan ucapan-perpisahan
ucapan-perpisahanMiz Jieha
 
Teks ucapan majlis perpisahan
Teks ucapan majlis perpisahanTeks ucapan majlis perpisahan
Teks ucapan majlis perpisahanjungle1982
 
Contoh karangan ucapan
Contoh karangan ucapanContoh karangan ucapan
Contoh karangan ucapanDorisca
 
MCQ test item analysis
MCQ test item analysisMCQ test item analysis
MCQ test item analysisSoha Rashed
 

En vedette (18)

ETAS_MCQ_05 dermatopathology
ETAS_MCQ_05 dermatopathologyETAS_MCQ_05 dermatopathology
ETAS_MCQ_05 dermatopathology
 
ETAS_MCQ_11 disorder of hair and nails
ETAS_MCQ_11 disorder of hair and nailsETAS_MCQ_11 disorder of hair and nails
ETAS_MCQ_11 disorder of hair and nails
 
ETAS_MCQ_10 manifestations of systemic diseases1
ETAS_MCQ_10 manifestations of systemic diseases1ETAS_MCQ_10 manifestations of systemic diseases1
ETAS_MCQ_10 manifestations of systemic diseases1
 
ETAS_MCQ_03 a genodermatoses
ETAS_MCQ_03 a genodermatosesETAS_MCQ_03 a genodermatoses
ETAS_MCQ_03 a genodermatoses
 
Teks ucapan menteri pelajaran malaysia perutusan 2011
Teks ucapan menteri pelajaran malaysia perutusan 2011Teks ucapan menteri pelajaran malaysia perutusan 2011
Teks ucapan menteri pelajaran malaysia perutusan 2011
 
Teks ucapan hari guru 2013
Teks ucapan hari guru 2013Teks ucapan hari guru 2013
Teks ucapan hari guru 2013
 
Dermatology without pics
Dermatology without picsDermatology without pics
Dermatology without pics
 
Teks ucapan wakil murid sempena hari guru
Teks ucapan wakil murid sempena hari guruTeks ucapan wakil murid sempena hari guru
Teks ucapan wakil murid sempena hari guru
 
Contoh Farewell Speech (ucapan/pidato perpisahan)
Contoh Farewell Speech (ucapan/pidato perpisahan)Contoh Farewell Speech (ucapan/pidato perpisahan)
Contoh Farewell Speech (ucapan/pidato perpisahan)
 
41499971 teks-ucapan-majlis-persaraan-guru-besar
41499971 teks-ucapan-majlis-persaraan-guru-besar41499971 teks-ucapan-majlis-persaraan-guru-besar
41499971 teks-ucapan-majlis-persaraan-guru-besar
 
SELAMAT BERSARA
SELAMAT BERSARASELAMAT BERSARA
SELAMAT BERSARA
 
Phototherapy treatment protocol
Phototherapy treatment protocolPhototherapy treatment protocol
Phototherapy treatment protocol
 
ucapan-perpisahan
 ucapan-perpisahan ucapan-perpisahan
ucapan-perpisahan
 
Teks ucapan majlis perpisahan
Teks ucapan majlis perpisahanTeks ucapan majlis perpisahan
Teks ucapan majlis perpisahan
 
Contoh karangan ucapan
Contoh karangan ucapanContoh karangan ucapan
Contoh karangan ucapan
 
Mrcp 2 dermatology
Mrcp 2 dermatologyMrcp 2 dermatology
Mrcp 2 dermatology
 
Koleksi pantun majlis
Koleksi pantun majlisKoleksi pantun majlis
Koleksi pantun majlis
 
MCQ test item analysis
MCQ test item analysisMCQ test item analysis
MCQ test item analysis
 

Similaire à ETAS_MCQ_15 dermatologic and cosmetic surgery

Lesions of oral cavity
Lesions of oral cavityLesions of oral cavity
Lesions of oral cavityAlkaKapil
 
Which of the following is an indication of a successful allogenic tooth implant
Which of the following is an indication of a successful allogenic tooth implantWhich of the following is an indication of a successful allogenic tooth implant
Which of the following is an indication of a successful allogenic tooth implantLama K Banna
 
Treatment of Osteoradionecrosis - DR Naveen (BDS)
Treatment of Osteoradionecrosis - DR Naveen (BDS)Treatment of Osteoradionecrosis - DR Naveen (BDS)
Treatment of Osteoradionecrosis - DR Naveen (BDS)Naveen Gokul Dr
 
emergency-nursing
 emergency-nursing emergency-nursing
emergency-nursingAnjo Ortiz
 
journal presentation on traumatic Tympanic membrane perforations.pptx
journal presentation on traumatic Tympanic membrane perforations.pptxjournal presentation on traumatic Tympanic membrane perforations.pptx
journal presentation on traumatic Tympanic membrane perforations.pptxAkanshaVerma97
 
Reversing a facial artery dermal filler occlusion
Reversing a facial artery dermal filler occlusion Reversing a facial artery dermal filler occlusion
Reversing a facial artery dermal filler occlusion Dr. Patrick J. Treacy
 
JC AMNIOTIC MEMEBRAME IN PREPROSTHETIC SURGERY
JC AMNIOTIC MEMEBRAME IN PREPROSTHETIC SURGERYJC AMNIOTIC MEMEBRAME IN PREPROSTHETIC SURGERY
JC AMNIOTIC MEMEBRAME IN PREPROSTHETIC SURGERYMalaM67
 
Facial danger zones techniques to maximize safety during soft tissue filler i...
Facial danger zones techniques to maximize safety during soft tissue filler i...Facial danger zones techniques to maximize safety during soft tissue filler i...
Facial danger zones techniques to maximize safety during soft tissue filler i...Nhat Nguyen
 
graft &flap physical therapy interventions.pptx
graft &flap physical therapy interventions.pptxgraft &flap physical therapy interventions.pptx
graft &flap physical therapy interventions.pptxMostafaAhmed891986
 
Vascular graft infection do we need antimicrobial grafts
Vascular graft infection do we need antimicrobial graftsVascular graft infection do we need antimicrobial grafts
Vascular graft infection do we need antimicrobial graftsuvcd
 
Acute care of facial burns (7th august 2010)
Acute care of  facial burns (7th august 2010)Acute care of  facial burns (7th august 2010)
Acute care of facial burns (7th august 2010)Tauseef Hassan
 
Comparison of Vacuum Assisted Closure Vs Conventional Moist Dressing in the M...
Comparison of Vacuum Assisted Closure Vs Conventional Moist Dressing in the M...Comparison of Vacuum Assisted Closure Vs Conventional Moist Dressing in the M...
Comparison of Vacuum Assisted Closure Vs Conventional Moist Dressing in the M...Priyatham Kasaraneni
 

Similaire à ETAS_MCQ_15 dermatologic and cosmetic surgery (20)

Lesions of oral cavity
Lesions of oral cavityLesions of oral cavity
Lesions of oral cavity
 
Which of the following is an indication of a successful allogenic tooth implant
Which of the following is an indication of a successful allogenic tooth implantWhich of the following is an indication of a successful allogenic tooth implant
Which of the following is an indication of a successful allogenic tooth implant
 
Sino nasal malignancies
Sino nasal malignanciesSino nasal malignancies
Sino nasal malignancies
 
Initial Care of Burn patients
Initial Care of Burn patientsInitial Care of Burn patients
Initial Care of Burn patients
 
Treatment of Osteoradionecrosis - DR Naveen (BDS)
Treatment of Osteoradionecrosis - DR Naveen (BDS)Treatment of Osteoradionecrosis - DR Naveen (BDS)
Treatment of Osteoradionecrosis - DR Naveen (BDS)
 
emergency-nursing
 emergency-nursing emergency-nursing
emergency-nursing
 
journal presentation on traumatic Tympanic membrane perforations.pptx
journal presentation on traumatic Tympanic membrane perforations.pptxjournal presentation on traumatic Tympanic membrane perforations.pptx
journal presentation on traumatic Tympanic membrane perforations.pptx
 
Reversing a facial artery dermal filler occlusion
Reversing a facial artery dermal filler occlusion Reversing a facial artery dermal filler occlusion
Reversing a facial artery dermal filler occlusion
 
134th publication sjm- 7th name
134th publication  sjm- 7th name134th publication  sjm- 7th name
134th publication sjm- 7th name
 
JC AMNIOTIC MEMEBRAME IN PREPROSTHETIC SURGERY
JC AMNIOTIC MEMEBRAME IN PREPROSTHETIC SURGERYJC AMNIOTIC MEMEBRAME IN PREPROSTHETIC SURGERY
JC AMNIOTIC MEMEBRAME IN PREPROSTHETIC SURGERY
 
Otoplasty: New Modification of the Mustardé technique
Otoplasty: New Modification of the Mustardé techniqueOtoplasty: New Modification of the Mustardé technique
Otoplasty: New Modification of the Mustardé technique
 
Fasciotomy Wound Closure.pdf
Fasciotomy Wound Closure.pdfFasciotomy Wound Closure.pdf
Fasciotomy Wound Closure.pdf
 
Facial danger zones techniques to maximize safety during soft tissue filler i...
Facial danger zones techniques to maximize safety during soft tissue filler i...Facial danger zones techniques to maximize safety during soft tissue filler i...
Facial danger zones techniques to maximize safety during soft tissue filler i...
 
graft &flap physical therapy interventions.pptx
graft &flap physical therapy interventions.pptxgraft &flap physical therapy interventions.pptx
graft &flap physical therapy interventions.pptx
 
Submental Intubation Technique for Airway during Surgery of Midfacial and Pan...
Submental Intubation Technique for Airway during Surgery of Midfacial and Pan...Submental Intubation Technique for Airway during Surgery of Midfacial and Pan...
Submental Intubation Technique for Airway during Surgery of Midfacial and Pan...
 
Rao Abu Bakar 2017009.docx
Rao Abu Bakar 2017009.docxRao Abu Bakar 2017009.docx
Rao Abu Bakar 2017009.docx
 
Vascular graft infection do we need antimicrobial grafts
Vascular graft infection do we need antimicrobial graftsVascular graft infection do we need antimicrobial grafts
Vascular graft infection do we need antimicrobial grafts
 
Total maxillectomy
Total maxillectomyTotal maxillectomy
Total maxillectomy
 
Acute care of facial burns (7th august 2010)
Acute care of  facial burns (7th august 2010)Acute care of  facial burns (7th august 2010)
Acute care of facial burns (7th august 2010)
 
Comparison of Vacuum Assisted Closure Vs Conventional Moist Dressing in the M...
Comparison of Vacuum Assisted Closure Vs Conventional Moist Dressing in the M...Comparison of Vacuum Assisted Closure Vs Conventional Moist Dressing in the M...
Comparison of Vacuum Assisted Closure Vs Conventional Moist Dressing in the M...
 

Plus de Derma202

Histopathplogical photos
Histopathplogical photosHistopathplogical photos
Histopathplogical photosDerma202
 
Slide study from ETAS
Slide  study from ETASSlide  study from ETAS
Slide study from ETASDerma202
 
Arab board primary exam in dermatology 2012
Arab board primary exam  in dermatology 2012Arab board primary exam  in dermatology 2012
Arab board primary exam in dermatology 2012Derma202
 
Dermatology
DermatologyDermatology
DermatologyDerma202
 
ETAS_MCQ_14 plants and creatures of dermatologic significance
ETAS_MCQ_14 plants and creatures of dermatologic significanceETAS_MCQ_14 plants and creatures of dermatologic significance
ETAS_MCQ_14 plants and creatures of dermatologic significanceDerma202
 
ETAS_MCQ_13 photobiology and photosensitivity disorders
ETAS_MCQ_13 photobiology and photosensitivity disordersETAS_MCQ_13 photobiology and photosensitivity disorders
ETAS_MCQ_13 photobiology and photosensitivity disordersDerma202
 
ETAS_MCQ_12 bullous diseases and blistering
ETAS_MCQ_12 bullous diseases and blisteringETAS_MCQ_12 bullous diseases and blistering
ETAS_MCQ_12 bullous diseases and blisteringDerma202
 
ETAS_MCQ_03 b genodermatoses
ETAS_MCQ_03 b genodermatosesETAS_MCQ_03 b genodermatoses
ETAS_MCQ_03 b genodermatosesDerma202
 
Derm handbook for medical students and junior doctors 2010
Derm handbook for medical students and junior doctors 2010Derm handbook for medical students and junior doctors 2010
Derm handbook for medical students and junior doctors 2010Derma202
 

Plus de Derma202 (9)

Histopathplogical photos
Histopathplogical photosHistopathplogical photos
Histopathplogical photos
 
Slide study from ETAS
Slide  study from ETASSlide  study from ETAS
Slide study from ETAS
 
Arab board primary exam in dermatology 2012
Arab board primary exam  in dermatology 2012Arab board primary exam  in dermatology 2012
Arab board primary exam in dermatology 2012
 
Dermatology
DermatologyDermatology
Dermatology
 
ETAS_MCQ_14 plants and creatures of dermatologic significance
ETAS_MCQ_14 plants and creatures of dermatologic significanceETAS_MCQ_14 plants and creatures of dermatologic significance
ETAS_MCQ_14 plants and creatures of dermatologic significance
 
ETAS_MCQ_13 photobiology and photosensitivity disorders
ETAS_MCQ_13 photobiology and photosensitivity disordersETAS_MCQ_13 photobiology and photosensitivity disorders
ETAS_MCQ_13 photobiology and photosensitivity disorders
 
ETAS_MCQ_12 bullous diseases and blistering
ETAS_MCQ_12 bullous diseases and blisteringETAS_MCQ_12 bullous diseases and blistering
ETAS_MCQ_12 bullous diseases and blistering
 
ETAS_MCQ_03 b genodermatoses
ETAS_MCQ_03 b genodermatosesETAS_MCQ_03 b genodermatoses
ETAS_MCQ_03 b genodermatoses
 
Derm handbook for medical students and junior doctors 2010
Derm handbook for medical students and junior doctors 2010Derm handbook for medical students and junior doctors 2010
Derm handbook for medical students and junior doctors 2010
 

Dernier

Mumbai Call Girls Service 9910780858 Real Russian Girls Looking Models
Mumbai Call Girls Service 9910780858 Real Russian Girls Looking ModelsMumbai Call Girls Service 9910780858 Real Russian Girls Looking Models
Mumbai Call Girls Service 9910780858 Real Russian Girls Looking Modelssonalikaur4
 
Russian Call Girls Chickpet - 7001305949 Booking and charges genuine rate for...
Russian Call Girls Chickpet - 7001305949 Booking and charges genuine rate for...Russian Call Girls Chickpet - 7001305949 Booking and charges genuine rate for...
Russian Call Girls Chickpet - 7001305949 Booking and charges genuine rate for...narwatsonia7
 
Call Girls Hsr Layout Just Call 7001305949 Top Class Call Girl Service Available
Call Girls Hsr Layout Just Call 7001305949 Top Class Call Girl Service AvailableCall Girls Hsr Layout Just Call 7001305949 Top Class Call Girl Service Available
Call Girls Hsr Layout Just Call 7001305949 Top Class Call Girl Service Availablenarwatsonia7
 
Kolkata Call Girls Services 9907093804 @24x7 High Class Babes Here Call Now
Kolkata Call Girls Services 9907093804 @24x7 High Class Babes Here Call NowKolkata Call Girls Services 9907093804 @24x7 High Class Babes Here Call Now
Kolkata Call Girls Services 9907093804 @24x7 High Class Babes Here Call NowNehru place Escorts
 
Call Girls Jayanagar Just Call 7001305949 Top Class Call Girl Service Available
Call Girls Jayanagar Just Call 7001305949 Top Class Call Girl Service AvailableCall Girls Jayanagar Just Call 7001305949 Top Class Call Girl Service Available
Call Girls Jayanagar Just Call 7001305949 Top Class Call Girl Service Availablenarwatsonia7
 
VIP Call Girls Lucknow Nandini 7001305949 Independent Escort Service Lucknow
VIP Call Girls Lucknow Nandini 7001305949 Independent Escort Service LucknowVIP Call Girls Lucknow Nandini 7001305949 Independent Escort Service Lucknow
VIP Call Girls Lucknow Nandini 7001305949 Independent Escort Service Lucknownarwatsonia7
 
Call Girl Service Bidadi - For 7001305949 Cheap & Best with original Photos
Call Girl Service Bidadi - For 7001305949 Cheap & Best with original PhotosCall Girl Service Bidadi - For 7001305949 Cheap & Best with original Photos
Call Girl Service Bidadi - For 7001305949 Cheap & Best with original Photosnarwatsonia7
 
Call Girls Service in Virugambakkam - 7001305949 | 24x7 Service Available Nea...
Call Girls Service in Virugambakkam - 7001305949 | 24x7 Service Available Nea...Call Girls Service in Virugambakkam - 7001305949 | 24x7 Service Available Nea...
Call Girls Service in Virugambakkam - 7001305949 | 24x7 Service Available Nea...Nehru place Escorts
 
Call Girls ITPL Just Call 7001305949 Top Class Call Girl Service Available
Call Girls ITPL Just Call 7001305949 Top Class Call Girl Service AvailableCall Girls ITPL Just Call 7001305949 Top Class Call Girl Service Available
Call Girls ITPL Just Call 7001305949 Top Class Call Girl Service Availablenarwatsonia7
 
Call Girls Thane Just Call 9910780858 Get High Class Call Girls Service
Call Girls Thane Just Call 9910780858 Get High Class Call Girls ServiceCall Girls Thane Just Call 9910780858 Get High Class Call Girls Service
Call Girls Thane Just Call 9910780858 Get High Class Call Girls Servicesonalikaur4
 
call girls in green park DELHI 🔝 >༒9540349809 🔝 genuine Escort Service 🔝✔️✔️
call girls in green park  DELHI 🔝 >༒9540349809 🔝 genuine Escort Service 🔝✔️✔️call girls in green park  DELHI 🔝 >༒9540349809 🔝 genuine Escort Service 🔝✔️✔️
call girls in green park DELHI 🔝 >༒9540349809 🔝 genuine Escort Service 🔝✔️✔️saminamagar
 
Call Girls Service Noida Maya 9711199012 Independent Escort Service Noida
Call Girls Service Noida Maya 9711199012 Independent Escort Service NoidaCall Girls Service Noida Maya 9711199012 Independent Escort Service Noida
Call Girls Service Noida Maya 9711199012 Independent Escort Service NoidaPooja Gupta
 
Call Girls Hosur Just Call 7001305949 Top Class Call Girl Service Available
Call Girls Hosur Just Call 7001305949 Top Class Call Girl Service AvailableCall Girls Hosur Just Call 7001305949 Top Class Call Girl Service Available
Call Girls Hosur Just Call 7001305949 Top Class Call Girl Service Availablenarwatsonia7
 
High Profile Call Girls Mavalli - 7001305949 | 24x7 Service Available Near Me
High Profile Call Girls Mavalli - 7001305949 | 24x7 Service Available Near MeHigh Profile Call Girls Mavalli - 7001305949 | 24x7 Service Available Near Me
High Profile Call Girls Mavalli - 7001305949 | 24x7 Service Available Near Menarwatsonia7
 
Noida Sector 135 Call Girls ( 9873940964 ) Book Hot And Sexy Girls In A Few C...
Noida Sector 135 Call Girls ( 9873940964 ) Book Hot And Sexy Girls In A Few C...Noida Sector 135 Call Girls ( 9873940964 ) Book Hot And Sexy Girls In A Few C...
Noida Sector 135 Call Girls ( 9873940964 ) Book Hot And Sexy Girls In A Few C...rajnisinghkjn
 
Low Rate Call Girls Mumbai Suman 9910780858 Independent Escort Service Mumbai
Low Rate Call Girls Mumbai Suman 9910780858 Independent Escort Service MumbaiLow Rate Call Girls Mumbai Suman 9910780858 Independent Escort Service Mumbai
Low Rate Call Girls Mumbai Suman 9910780858 Independent Escort Service Mumbaisonalikaur4
 
Russian Call Girls Gunjur Mugalur Road : 7001305949 High Profile Model Escort...
Russian Call Girls Gunjur Mugalur Road : 7001305949 High Profile Model Escort...Russian Call Girls Gunjur Mugalur Road : 7001305949 High Profile Model Escort...
Russian Call Girls Gunjur Mugalur Road : 7001305949 High Profile Model Escort...narwatsonia7
 
Call Girls Hebbal Just Call 7001305949 Top Class Call Girl Service Available
Call Girls Hebbal Just Call 7001305949 Top Class Call Girl Service AvailableCall Girls Hebbal Just Call 7001305949 Top Class Call Girl Service Available
Call Girls Hebbal Just Call 7001305949 Top Class Call Girl Service Availablenarwatsonia7
 
Glomerular Filtration and determinants of glomerular filtration .pptx
Glomerular Filtration and  determinants of glomerular filtration .pptxGlomerular Filtration and  determinants of glomerular filtration .pptx
Glomerular Filtration and determinants of glomerular filtration .pptxDr.Nusrat Tariq
 
Call Girl Koramangala | 7001305949 At Low Cost Cash Payment Booking
Call Girl Koramangala | 7001305949 At Low Cost Cash Payment BookingCall Girl Koramangala | 7001305949 At Low Cost Cash Payment Booking
Call Girl Koramangala | 7001305949 At Low Cost Cash Payment Bookingnarwatsonia7
 

Dernier (20)

Mumbai Call Girls Service 9910780858 Real Russian Girls Looking Models
Mumbai Call Girls Service 9910780858 Real Russian Girls Looking ModelsMumbai Call Girls Service 9910780858 Real Russian Girls Looking Models
Mumbai Call Girls Service 9910780858 Real Russian Girls Looking Models
 
Russian Call Girls Chickpet - 7001305949 Booking and charges genuine rate for...
Russian Call Girls Chickpet - 7001305949 Booking and charges genuine rate for...Russian Call Girls Chickpet - 7001305949 Booking and charges genuine rate for...
Russian Call Girls Chickpet - 7001305949 Booking and charges genuine rate for...
 
Call Girls Hsr Layout Just Call 7001305949 Top Class Call Girl Service Available
Call Girls Hsr Layout Just Call 7001305949 Top Class Call Girl Service AvailableCall Girls Hsr Layout Just Call 7001305949 Top Class Call Girl Service Available
Call Girls Hsr Layout Just Call 7001305949 Top Class Call Girl Service Available
 
Kolkata Call Girls Services 9907093804 @24x7 High Class Babes Here Call Now
Kolkata Call Girls Services 9907093804 @24x7 High Class Babes Here Call NowKolkata Call Girls Services 9907093804 @24x7 High Class Babes Here Call Now
Kolkata Call Girls Services 9907093804 @24x7 High Class Babes Here Call Now
 
Call Girls Jayanagar Just Call 7001305949 Top Class Call Girl Service Available
Call Girls Jayanagar Just Call 7001305949 Top Class Call Girl Service AvailableCall Girls Jayanagar Just Call 7001305949 Top Class Call Girl Service Available
Call Girls Jayanagar Just Call 7001305949 Top Class Call Girl Service Available
 
VIP Call Girls Lucknow Nandini 7001305949 Independent Escort Service Lucknow
VIP Call Girls Lucknow Nandini 7001305949 Independent Escort Service LucknowVIP Call Girls Lucknow Nandini 7001305949 Independent Escort Service Lucknow
VIP Call Girls Lucknow Nandini 7001305949 Independent Escort Service Lucknow
 
Call Girl Service Bidadi - For 7001305949 Cheap & Best with original Photos
Call Girl Service Bidadi - For 7001305949 Cheap & Best with original PhotosCall Girl Service Bidadi - For 7001305949 Cheap & Best with original Photos
Call Girl Service Bidadi - For 7001305949 Cheap & Best with original Photos
 
Call Girls Service in Virugambakkam - 7001305949 | 24x7 Service Available Nea...
Call Girls Service in Virugambakkam - 7001305949 | 24x7 Service Available Nea...Call Girls Service in Virugambakkam - 7001305949 | 24x7 Service Available Nea...
Call Girls Service in Virugambakkam - 7001305949 | 24x7 Service Available Nea...
 
Call Girls ITPL Just Call 7001305949 Top Class Call Girl Service Available
Call Girls ITPL Just Call 7001305949 Top Class Call Girl Service AvailableCall Girls ITPL Just Call 7001305949 Top Class Call Girl Service Available
Call Girls ITPL Just Call 7001305949 Top Class Call Girl Service Available
 
Call Girls Thane Just Call 9910780858 Get High Class Call Girls Service
Call Girls Thane Just Call 9910780858 Get High Class Call Girls ServiceCall Girls Thane Just Call 9910780858 Get High Class Call Girls Service
Call Girls Thane Just Call 9910780858 Get High Class Call Girls Service
 
call girls in green park DELHI 🔝 >༒9540349809 🔝 genuine Escort Service 🔝✔️✔️
call girls in green park  DELHI 🔝 >༒9540349809 🔝 genuine Escort Service 🔝✔️✔️call girls in green park  DELHI 🔝 >༒9540349809 🔝 genuine Escort Service 🔝✔️✔️
call girls in green park DELHI 🔝 >༒9540349809 🔝 genuine Escort Service 🔝✔️✔️
 
Call Girls Service Noida Maya 9711199012 Independent Escort Service Noida
Call Girls Service Noida Maya 9711199012 Independent Escort Service NoidaCall Girls Service Noida Maya 9711199012 Independent Escort Service Noida
Call Girls Service Noida Maya 9711199012 Independent Escort Service Noida
 
Call Girls Hosur Just Call 7001305949 Top Class Call Girl Service Available
Call Girls Hosur Just Call 7001305949 Top Class Call Girl Service AvailableCall Girls Hosur Just Call 7001305949 Top Class Call Girl Service Available
Call Girls Hosur Just Call 7001305949 Top Class Call Girl Service Available
 
High Profile Call Girls Mavalli - 7001305949 | 24x7 Service Available Near Me
High Profile Call Girls Mavalli - 7001305949 | 24x7 Service Available Near MeHigh Profile Call Girls Mavalli - 7001305949 | 24x7 Service Available Near Me
High Profile Call Girls Mavalli - 7001305949 | 24x7 Service Available Near Me
 
Noida Sector 135 Call Girls ( 9873940964 ) Book Hot And Sexy Girls In A Few C...
Noida Sector 135 Call Girls ( 9873940964 ) Book Hot And Sexy Girls In A Few C...Noida Sector 135 Call Girls ( 9873940964 ) Book Hot And Sexy Girls In A Few C...
Noida Sector 135 Call Girls ( 9873940964 ) Book Hot And Sexy Girls In A Few C...
 
Low Rate Call Girls Mumbai Suman 9910780858 Independent Escort Service Mumbai
Low Rate Call Girls Mumbai Suman 9910780858 Independent Escort Service MumbaiLow Rate Call Girls Mumbai Suman 9910780858 Independent Escort Service Mumbai
Low Rate Call Girls Mumbai Suman 9910780858 Independent Escort Service Mumbai
 
Russian Call Girls Gunjur Mugalur Road : 7001305949 High Profile Model Escort...
Russian Call Girls Gunjur Mugalur Road : 7001305949 High Profile Model Escort...Russian Call Girls Gunjur Mugalur Road : 7001305949 High Profile Model Escort...
Russian Call Girls Gunjur Mugalur Road : 7001305949 High Profile Model Escort...
 
Call Girls Hebbal Just Call 7001305949 Top Class Call Girl Service Available
Call Girls Hebbal Just Call 7001305949 Top Class Call Girl Service AvailableCall Girls Hebbal Just Call 7001305949 Top Class Call Girl Service Available
Call Girls Hebbal Just Call 7001305949 Top Class Call Girl Service Available
 
Glomerular Filtration and determinants of glomerular filtration .pptx
Glomerular Filtration and  determinants of glomerular filtration .pptxGlomerular Filtration and  determinants of glomerular filtration .pptx
Glomerular Filtration and determinants of glomerular filtration .pptx
 
Call Girl Koramangala | 7001305949 At Low Cost Cash Payment Booking
Call Girl Koramangala | 7001305949 At Low Cost Cash Payment BookingCall Girl Koramangala | 7001305949 At Low Cost Cash Payment Booking
Call Girl Koramangala | 7001305949 At Low Cost Cash Payment Booking
 

ETAS_MCQ_15 dermatologic and cosmetic surgery

  • 1. Dermatologic and Cosmetic Surgery 1) The tensile strength of a wound 2 weeks after surgery compared to intact skin is: A. 10%Correct Choice B. 50% C. 30% D. 70% E. 100% The tensile strength of a wound approaches 10% of normal skin strength at 2 weeks postoperatively. Wound healing is a process which takes many months, yet the tensile strength of a wound never exceeds 70-80% of the tensile strength of intact skin. The use of subcuticular sutures is therefore critical to minimize the tension on a healing wound and prevent scar widening 2) What risk does an organ transplant recipient have of developing melanoma? A. Same risk as general population B. Less risk than general population C. 10- fold increased risk D. 50-fold increased risk E. 3-fold increased riskCorrect Choice Organ transplant recipients are at risk for developing many cutaneous tumors. The risk of melanoma is 3-fold that of the general population whereas these patients have a 65-fold increased risk of developing squamous cell carcinoma 3) A Z- plasty is performed to: A. Reorient a scar Correct Choice B. Make use of skin from an area of laxity C. Obtain wound eversion D. Shorten the length of an excision E. Close a wound on a convex surface A Z-plasty is a type of transposition flap that may be used to reorient a scar, especially when the scar crosses skin tension lines. It is also used to change the scar length or vector and ease scar contractures 4) A Mohs surgery fellow experiences burning and tingling of her fingertips 6 months after starting her fellowship. She most likely has been exposed to which one of the following chemicals? A. Chlorhexidine gluconate B. Povodine-iodine C. Hematoxylin D. Toluidine blue E. Hexachlorophene Correct Choice 1
  • 2. Hexachlorophene is an antibacterial agent that was first introduced in 1944. Its use was discontinued in the 1970s when it was discovered to have neurotoxic side effects 5) The role of imiquimod in the treatment of superficial basal cell carcinoma is based on the induction of which cytokine? A. Tumor growth factor-alpha B. Interferon-gamma Correct Choice C. Tumor necrosis factor–beta D. Interleukin-2 E. Interleukin-4 Imiquimod is an immunomodulator which induces the production of cytokines that are involved in cell-mediated immunity. These cytokines include interferon-alpha, interferon-gamma, interleukin-1, interleukin-10 and tumor necrosis factor-alpha. Induction of these cytokines results in antiviral and antitumor activity in vivo 6) All of the following are branches of the internal carotid artery except: A. Ophthalmic artery B. Supratrochlear artery C. Supraorbital artery D. Angular arteryCorrect Choice E. Dorsal nasal artery Branches of the internal carotid artery include the opthamic artery, supraorbital artery, dorsal nasal artery, supratrochlear artery, palpebral artery, and lacrimal artery. Branches of the external carotid artery include the superficial temporal artery, facial artery (inferior labial, superior labial, angular branches), maxillary artery, posterior auricular, and occipital artery. The external and internal carotid arteries anastamose in the periorbital region 7) How long after cutaneous infiltration of lidocaine with epinephrine is maximal vasoconstriction achieved? A. 15 minutes Correct Choice B. 7 minutes C. 1 minute D. 30 minutes E. 1 hour The use of epinephrine with local anesthesia has two main advantages. Firstly, the epinephrine is a vasoconstrictor and assists in controlling bleeding during surgery. Secondly, and also a direct result of its vasoconstrictive effects, epinephrine helps prolong the duration of the anesthetic agent 100% to 200% by delaying its absorption from the surgical site. Although the anesthetic properties of lidocaine take effect within the first few minutes of infiltration, the vasoconstrictive effects of epinephrine require approximately 15 minutes to fully develop 8) Which of the following topical antibacterial agents demonstrates activity against Pseudomonas species? 2
  • 3. A. Erythromycin B. Mupirocin C. Polymyxin Correct Choice D. Bacitracin E. Clindamycin Polymyxins are peptides produced by the organism Bacillus polymyxa. They are bactericidal against Pseudomonas aeruginosa, Proteus mirabilis, Enterobacter, Klebsiella and Escherichia coli. Since they provide little coverage against gram-positive organisms, polymyxins are often combined with other antibacterial agents to increase their spectrum of activity 9) Healing by secondary intention will offer the most favorable cosmetic result at which location? A. Malar cheek B. Nasal tip C. Chin D. Medial canthusCorrect Choice E. Forehead Indications for healing of full-thickness skin wounds by secondary intention include infected wounds, patients who refuse reconstructive surgery or who are poor surgical risks for reconstructive surgery, and resection of tumors with high rates of recurrence. In addition, secondary intention, or granulation, can be utilized when the cosmetic result is anticipated to be superior or equal to reconstruction. Disadvantages of healing by granulation include prolonged wound healing time and somewhat unpredictable cosmetic results. Anatomic sites lending themselves best to second intention healing, with excellent results, are the concave areas on the face. These include the medial canthus, the nose-cheek junction, the auricular concha, the preauricular cheek, and the retroauricular scalp. The nasal tip, malar cheek, chin and forehead are all convex surfaces on the face 10) Compared to intact skin, what is the tensile strength of a wound 6 months after surgery? A. 30% B. 90% C. 50% D. 70%Correct Choice E. 100% Tissue remodeling allows the host to develop a scar that is approximately 70% of the original strength of the skin 11) The following flaps are types of transposition flaps except: A. A to TCorrect Choice B. Nasolabial C. Bilobed D. Z plasty E. Rhombic flap 3
  • 4. The Z plasty, Bilobed, Rhombic, nasolabial flaps are all examples of transposition flaps. In each of these, the flap is transposed over a normal piece of skin to be placed in the recipient site. Whereas, the A to T flap, is an advancement flap, in which tissue is moved in a linear direction to cover the defect 12) Which of the following demonstrates the highest risk of metastasis? A. SCC within a scar Correct Choice B. SCC on the scalp C. SCC on the nose D. SCC on the ear E. SCC on the lip Location of tumor is an important risk factor for metastasis of squamous cell carcinoma. Compared with a 10% likelihood of metastasis for tumors located on the ear or the lip, an SCC developing in the scar, however, has been estimated to metastasize at a rate as high as 30-40% 13) A surgeon plans to repair a defect on the lower lip with a wedge resection. All of the following statements are true regarding wedge resection of the lower lip except? A. the vermillion border is marked prior to excision B. the repair involves a two layered closure of skin and mucosaCorrect Choice C. up to 1/3 of the lip can be excised and closed primarily D. the mental crease should not be crossed E. a V-shaped excision is often used The lower lip has such great elasticity that full-thickness lesions that involve up to 1/3 of the lip can be excised and repaired primarily with excellent cosmetic and functional results. This type of repair requires a three layered closure of mucosa, skin and muscle. Larrabee W. In: Principles of Facial Reconstruction. Lippincott-Raven: Philadelphia, 1995; 182-183 14) Endocarditis prophylaxis is recommended for each of the following except: A. Previous bacterial endocarditis B. Atrial septal defectsCorrect Choice C. Mitral valve prolapse with regurgitation D. Prosthetic valves E. Hypertrophic cardiomyopathy Preoperative antibiotics are recommended for endocarditits prophylaxis in those patients with prostheitc valves, congenital cardiac malformations, previous bacterial endocarditis, rheumatic heart disease with valve dysfunction, hypeertrophic cardiomyopathy, and mitral valve prolapse with refurgitaion. Classically 1 gram of Dicloxacillin or cephalexin is given 1 hour preoperatively and an additional 500 mg is given 6 hours post op. Clindamycin can be givenin those patients who are penicillin allergic. Preopertive antibiotics for endocarditis prohpylaxis are not required in those patients who have an Atrial Septal Defect (ASD). pacemaker, rheumatic fever without valve dysfunction, or history of Coronary Artery Bypass Graft (CABG 4
  • 5. 15) Which of the following is true regarding lidocaine: A. Allergy most commonly occurs to propylene glycol preservatives B. Maximum dose with no epinephrine is 7mg/kg C. Beta blockers increase lidocaine levelsCorrect Choice D. 1% lidocaine is equal to 1g/10ml E. Duration with no epinephrine is 4-6 hours Beta blockers increase lidocaine levels. The rest of the answers are false. 1% lidocaine is equal to 1g/100ml or 10mg/ml. Duruation of lidocaine with no epi is 30-60 minutes. Maximum dose of lidocaine with no epi is 4.5 mg/kg, with epi it is 7 mg/kg. Allergy to lidocaine is most commonly due to paraben preservatives 16) On average, how much does a full-thickness graft contract when removed from a donor site? A. 30% B. 15%Correct Choice C. 80% D. 50% E. 60% On the average, how much does a full-thickness graft may contract by about 15% when removed from a donor site 17) The surgical instrument shown in the photograph is a: A. Dermablade B. No. 10 blade C. No. 11 bladeCorrect Choice D. No. 15 blade E. Beaver blade Surgical blades are made from either carbon steel or stainless steel and may be coated with Teflon to reduce their drag through tissue. A no.11 blade, which is a narrow blade that ends in a sharp point, is used to make stab incisions and is often employed in incision and drainage procedures 18) True statements regarding skin cancer in organ transplant recipients include all of the following except: A. Mohs micrographic surgery indicated for in-transit metastases Correct Choice B. 65 fold increase in development of SCC compared with general population C. Extent of tumor development related to degree of immunosuppression D. Skin cancer is the most common cancer in transplant patients E. Cutaneous malignancies develop 3-5 years after organ transplantation In-transit metastases is a common manifestation of metastatic disease 19) Which of the following local anesthetics has the longest duration of action? 5
  • 6. A. Mepivacaine B. Benzocaine C. Lidocaine D. Procaine E. BupivacaineCorrect Choice Bupivacaine is an amide type of local anesthetic. Its duration of action is approximately 3-5 hours and is the longest one listed. Following bupivacaine is mepivacaine, lidocaine, and procaine. The duration of action is based on the amine portion of the molecule 20) Which needle shape is most commonly used in cutaneous surgery? A. 1/4 circle B. 5/8 circle C. 3/8 circleCorrect Choice D. compound curve E. 1/2 circle Needles are either straight or curved. Curved needles have their curvature described either as a fraction of a circle or a compound curve. The greater the fraction of a circle, the more pronation and supination of the wrist required by the surgeon to place the needle. The 3/8 circle needle is easy to use in large, superficial areas and is the most commonly used needle for cutaneous surgery 21) Which of the following is the earliest symptom of lidocaine toxicity: A. Perioral tinglingCorrect Choice B. Nystagmus C. Seizure D. Tachycardia E. Cyanosis The maximum dosage of lidocaine is 4.5 mg/kg without epinephrine and 7.0 mg/kg with epinephrine. Signs of lidocaine toxicity start at 1-5 microgm/ml with increased anxiety, talkativeness, tinnitus, numbess/tingling around lips, metallic taste, double vision. Higher levels of toxicity may cause nystagmus, muscle twitching, tremor and finally seizures and respiratory arrest 22) Which of the following cosmetic injectables does not illicit an inflammatory response? A. Artecoll (polymethylmethacrylate) B. Sculptra (poly L-lactic acid) C. Radiance (hydroxyapatite)Correct Choice D. Zyplast (collagen) E. Silicone Radiance is an injectable, biodegradable filler composed of calcium hydroxyapatite microspheres. It is a normal constituent of bone and is therefore biocompatible. Current studies are examining the role of hydroxyapatite in augmenting the craniofacial skeleton 6
  • 7. 23) Each of the following are branches of the internal carotid artery except: A. Ophthalmic artery B. Supraorbital artery C. Supratrochlear artery D. Angular artery Correct Choice E. Dorsal nasal artery The angular artery is a branch of the external carotid artery. It arises from the facial artery and courses superiorly along the nasofacial angle until it reaches the area of the medial canthal tendon. At this location, the angular artery anastamoses with the dorsal nasal branch of the ophthalmic artery establishing a communication between the internal and external carotid arterial systems 24) Which of the following parameters determines the wavelength of a laser? A. Fluence B. Spot size C. Lens length D. Medium Correct Choice E. Pulse duration Laser light is monochromatic light that is emitted at a single wavelength. The wavelength of the laser is determined by the medium in the optical cavity of the laser through which the light passes. The medium may be solid, liquid or gas 25) The cutaneous lip and chin are divided into cosmetic units by which anatomic boundary? A. Philtral ridge B. Mental creaseCorrect Choice C. Nasolabial fold D. Vermillion border E. Marionette lines The term cosmetic unit is used to define regions of the face by specific characteristics such as color, texture, presence of hair, and prominence of sebaceous glands. In dermatologic surgery, it is preferable to conceal incisions within the boundary lines of cosmetic units. The mental crease, as demonstrated in the photograph, divides the cutaneous lip and chin 26) Which statement is true regarding relaxed skin tension lines (RSTLs)? A. incisional scars should be placed perpendicular to RSTLs B. the long axis of a wound often lies in the direction of the RSTL Correct Choice C. they lie parallel to the underlying muscles D. they occur as a result of increased elastic tone E. smiling minimizes the appearance of RSTLs Relaxed skin tension lines are creases in the skin that are present at rest and develop as a consequence of the decreased elasticity that occurs with aging and solar damage. These lines lie perpendicular to the underlying musculature and can be accentuated by smiling or frowning. An 7
  • 8. undermined circular wound will often form an oval shape due to muscular tension, and will have its long axis oriented within these relaxed tension lines. Knowledge of RSTLs is important in cutaneous surgery because placing incisions within these lines will ensure the most favorable cosmetic result 27) Pre-testing for allergy to collagen is required for which cosmetic filler? A. Silicone B. Artecoll Correct Choice C. Radiance D. Perlane E. Cosmoderm Artecoll is a nonbiodegradable injectable filler composed of microspheres of polymethylmethacrylate which are suspended in bovine collagen. As with other types of injectable bovine collagen, patients must be tested for allergy to bovine collagen prior to treatment 28) Damage to the zygomatic branch of the facial nerve leads to which of the following: A. Facial asymmetry B. Eyelid ectropion and inability to close eyelidCorrect Choice C. Winged scapula D. Inability to pucker lips E. Unilateral eyelid ptosis Damage to the zygomatic nerve causes eyelid ectropion and inability to close eyelid 29) A patient has a 2.0 cm surgical defect on the left lower eyelid after having Mohs surgery. Which management option would be the least appropriate? A. Primary closure B. Secondary intentionCorrect Choice C. Rhombic flap D. Full-thickness skin graft E. Tripier flap In lower eyelid reconstruction, tension should be oriented parallel to the lower eyelid to prevent the development of ectropion. Secondary intention would be the least appropriate management option in this location because normal wound contraction would increase the risk of ectropion 30) Cosmetic units are defined by each of the following anatomic boundaries except: A. Nasofacial sulcus B. Glabellar region C. Philtral ridge D. Nasolabial folds E. Nasal tip Correct Choice 8
  • 9. The term cosmetic unit is used to define regions of the face by specific characteristics such as color, texture, presence of hair, and prominence of sebaceous glands. In dermatologic surgery, it is preferable to conceal incisions within the boundary lines of cosmetic units. In addition, moving skin from one cosmetic unit to another should be avoided when possible to minimize the appearance of apposed skin of dissimilar quality 31) The O to Z flap is what type of flap? A. Pedicle B. RotationCorrect Choice C. Transposition D. Bilobed E. Advancement The O to Z flap is a bilateral rotation flap in which tissue is moved from two ends to cover a central defect. It is often used on the scalp or lower extremities in locations where tissue laxity is not present. 32) A patient who had liposuction 5 weeks ago presents with multiple firm nodules at the cannula insertion sites. You suspect the diagnosis is: A. Herpes simplex infection B. Cold panniculitis C. Mycobacterial infectionCorrect Choice D. Foreign body granuloma E. Organized hematoma Atypical mycobacterial infections are occurring with increasing frequency after cosmetic surgery procedures. These infections typically occur 4-14 weeks after a procedure as a late-occurring complication. Firm nodules at the treatment site or dehiscence of a previously healed wound may be presenting signs of atypical mycobacterial infection 33) The main advantage of selecting 4-0 Vicryl rather than 5-0 Vicryl to suture a wound is: A. Smaller suture diameter B. Smaller needle C. Increased tensile strength Correct Choice D. Increased suture memory E. Increased knot security Sutures are classified according to the United States Pharmacopeia (USP) criteria. This classification system specifies the diameter of a given suture material that is required to produce a certain tensile strength. The smaller the cross-sectional diameter of a suture material, the higher the USP number that is assigned. Thus, 4-0 Vicryl will have greater tensile strength and a larger cross-sectional diameter than 5-0 Vicryl 34) All of the following statements are true regarding the ‘MCW Melanoma Cocktail’ except: A. it demonstrates micrometastases in sentinel lymph nodes 9
  • 10. B. it is performed intraoperatively C. it is a mixture of polyclonal antibodies to tyrosinaseCorrect Choice D. it is a mixture of monoclonal antibodies to MART-1 E. it is a mixture of monoclonal antibodies to Melan-A The ‘MCW Melanoma Cocktail’ is an immunostain made up of monoclonal antibodies to MART-1, Melan-A and tryrosine. It is used intraoperatively during sentinel lymph node biopsy and allows for rapid and accurate determination of micrometastases. Shidham VB et al. Optimization of an immunostaining protocol for the rapid intraoperative evaluation of melanoma sentinel lymph node imprint smears with the ‘MCW Melanoma Cocktail’. Cytojournal 2004;1(1):2 35) The “trapdoor effect” that can occur with a flap can be avoided by which of the following techniques: A. Using additional sutures B. Avoid thinning the flap C. Using a flap with length to width ratio of less than 3:1 D. Performing the flap in a two staged procedure E. Wide underminingCorrect Choice Wide undermining of the primary defect can release tension and prevent tenting or the trap door deformity of the flap 36)The nasolabial two stage flap uses depends on which artery for flap survival: A. Lateral Nasal artery B. Angular arteryCorrect Choice C. Superior labial artery D. Inferior labial artery E. Supratrochlear artery Angular artery provides blood supply for the nasolabial transposition flap 37) A patient with end stage liver disease comes to your surgical practice. Which of the following anesthetics should be used with this patient? A. Mepivicaine B. Lidocaine C. BenzocaineCorrect Choice D. Etidocaine E. Prilocaine Benzocaine, an ester, should be used as it is the only anesthetic mentioned above that is metabolized by the kidney and not the liver 10
  • 11. 38) A 40 year-old woman from Southern California has wrinkles at rest on her forehead, scattered telangiectasia on her nose and a few seborrheic keratoses on her chest. What category in Glogau's photoaging classification scale does this patient represent? A. Type 5 B. Type 2 C. Type 4 D. Type 1 E. Type 3 Correct Choice Glogau’s photoaging scale is a classification system that employs clinical markers of photodamage to determine an individual’s level of photoaging. The scale ranges from Type 1 through Type 4, with Type 4 demonstrating the most extensive photodamage. Type 3 on the Glogau scale is classified as advanced photoaging and describes a patient who is typically 50 years of age or older, has wrinkles at rest, telangiectasias, obvious dyschromias and visible keratoses 39) Which post-operative complication is shown in the photograph? A. Temporal nerve paralysis B. Ectropion Correct Choice C. Trap door deformity D. Hypertrophic scar E. Eclabium Ectropion occurs when the eyelid margin turns outward away from the eye. In lower eyelid reconstruction, tension should be oriented parallel to the lower eyelid to prevent the development of ectropion 40) Which of the following facial rejuvenation techniques creates microthermal zones? A. Radiofrequency B. Intense pulsed light C. Dermabrasion D. Fractional photothermolysisCorrect Choice E. CO2 laser Fractional photothermolysis utilizes an infrared laser to create thousands of microthermal zones with normal skin remaining between these zones. The zones of normal skin support rapid re- epithelialization and result in much faster healing times than with traditional resurfacing procedures 41) Preliminary studies with phosphatidylcholine demonstrate efficacy in the treatment of: A. Soft tissue augmentation B. Acne scarring C. Periorbital fat padsCorrect Choice D. Postinflammatory hyperpigmentation E. Superficial rhytids 11
  • 12. Phosphatidylcholine is a lecithin-derived phospholipid which induces lipolysis when injected into adipose tissue. Dissolution of fat is likely due to a detergent effect produced by the phosphatidylcholine 42) A patient with a deep vertical frown line in the glabellar region requests Botox. Treatment will mostly target which one of the following muscles? A. Orbicularis oculi B. Depressor superciliaris C. Orbicularis oculi D. Levator palpebrae superioris E. Corrugator superciliaris Correct Choice Although the procerus and orbicularis oculi muscles are contributory, the development of vertical glabellar frown lines is mostly due to the overactivity of the corrugator supercilaris. These muscles function to pull the eyebrow inferomedially 43) Botulinum toxin type A cleaves which protein in the presynaptic neuron? A. Acetylcholine B. Synaptobrevin (VAMP) C. Snap-25Correct Choice D. Serotonin E. Syntaxin Botulinum toxin type A is FDA approved for the treatment of glabellar wrinkles. Botulinum toxins act by a three step process of binding, internalization by receptor medicated endocytosis and enzymatic activation. It has specific light chain intracellular binding sites and different sites of action on different SNARE (synaptosomal associated protein receptor [SNAP]) proteins. The SNARE proteins are intimately involved in releasing acetylcholine at presynaptic terminals. Botulinum toxin A cleaves SNAP-25 whereas B cleaves synaptobrevin 44) In a patient who is allergic to paraphenylinediamine, which of the following anesthetics should be avoided? A. BenzocaineCorrect Choice B. Bupivicaine C. Prilocaine D. Mepivicaine E. Etidocaine Paraphenyelenediamine is a common allergen found in permanent hair dyes. In individuals who are allergic to paraphenylendiamine, may also have allergic reactions to ester anesthetics, like benzocaine. The other answer choices are amide anesthetics. Other crossreactants to paraphenyelediamine include the preservative parabens, sulfonamides, and thiazide diuretics 45) The anatomic structure identified in the photograph is called the: A. Philtral ridge 12
  • 13. B. Nasal groove C. Nasal ala D. Soft triangle E. ColumellaCorrect Choice The columella is one of the cosmetic subunits of the nose. It is formed by the medial crura of the alar cartilage 46) All of the following thrombotic complications have been reported after discontinuing aspirin therapy prior to surgery except: A. Stroke B. Transient ischemic attack C. Pulmonary embolismCorrect Choice D. Cerebral embolism E. Myocardial infarction Kovich et al. reported thrombotic complications in patients who had stopped either coumadin or aspirin peri-operatively. Thrombotic complications in patients who stopped warfarin included stroke, TIA, myocardial infarction, cerebral embolism, death, DVT, pulmonary embolus, and blindness. Thrombotic complications in patients who stopped aspirin included stroke, TIA, myocardial infarction, cerebral embolism and death. No DVT or pulmonary embolus was reported 47) Which of the following immunosuppressive agents may exert a protective effect against the development of skin cancers in organ transplant patients? A. RapamycinCorrect Choice B. Tacrolimus C. Corticosteroids D. Cyclosporine E. Azathioprine Rapamycin (also called sirolimus) is a macrolide antibiotic and a structural analog of FK 506. It is a potent immunosuppressive agent which inhibits mTOR (a member of P13K family kinases). Despite its immunosuppressive effects, preliminary data show a decreased incidence of skin cancer in organ transplant patients treated with rapamycin and postulate that it may exert a protective effect against cutaneous malignancies 48) Injection of Botox at the location identified in the photograph would denervate which muscle? A. Frontalis B. Corrugator superciliaris C. Levator palpebrae superioris D. Orbicularis oculi E. ProcerusCorrect Choice The development of vertical glabellar frown lines is due to the overactivity of the orbicularis oculi, procerus and corrugator supercilaris muscles. These muscles function to pull the eyebrow inferomedially 13
  • 14. 49) A safe dose of lidocaine with epinephrine when used in tumescent anesthesia is: A. 3 mg/kg B. 7 mg/kg C. 4.5 mg/kg D. 20 mg/kg E. 50 mg/kg Correct Choice The maximum recommended dosage of lidocaine in adults is 4.5 mg/kg without epinephrine, 7.0 mg/kg with epinephrine, and 55 mg/kg in tumescent anesthesia for liposuction 50) Which of the following determines the wavelength of a laser? A. Spot size B. Medium Correct Choice C. Pulse duration D. Fluence E. Q switch The wavelength of a laser is determined by the medium 51) Which of the following cosmetic injectables has the longest duration of action? A. Sculptra (poly L-lactic acid)Correct Choice B. Restylane (hyaluronic acid) C. Cosmoderm (collagen) D. Myobloc E. Botox Sculptra (called New-Fill outside of the US) is a biodegradable filler composed of poly-L-lactic acid, the same material used in absorbable sutures. Preliminary studies have demonstrated longevity of the filler at two years post-treatment. Valantin MA et al. Polylactic acid implants (New-Fill) to correct facial lipoatrophy in HIV-infected patients: results of the open-label study VEGA. AIDS. 2003;17:2471-2477 52) Which is the most important criterion for performing Mohs micrographic surgery? A. Aggressive histology B. Size of tumor > 2cm C. Tumor recurrence D. Tumor is contiguous Correct Choice E. Location of tumor Tumor that grows as a contiguous mass is an absolute requirement for Mohs micrographic surgery. If a tumor exhibits discontiguous growth, discrete foci may be missed in the thin tissue layers and thereby lead to a false-negative pathology interpretation. Size, location, recurrence and histology are all indications for Mohs micrographic surgery rather than absolute requirements 14
  • 15. 53) The use of topical vitamin K has been shown to: A. Induce keratinocyte differentiation B. Increase collagen production C. Minimize the appearance of telangiectasia D. Reduce the severity of laser-induced purpura Correct Choice E. Decrease the appearance of infraorbital pigmentation Treatment of benign vascular lesions with the pulsed dye laser often produces significant postoperative purpura. Topical vitamin K has been shown to decrease the severity of laser-induced purpura although its mechanism of action has yet to be determined. No other cosmetic effects of topical vitamin K have been proven to be statistically significant 54) Which of the following is the major antioxidant in the human epidermis? A. Ascorbic acid B. Coenzyme Q10 C. Alpha-tocopherol Correct Choice D. Superoxide dismutase E. Glutathione Research into the preventive role of topical antioxidants in photoaging is based on the free radical theory of aging. All of the choices above are antioxidants which are naturally found in the skin, however, alpha-tocopherol is the major antioxidant in human epidermis. Epidermal depletion of this vitamin has been shown to be an early and sensitive marker of environmental oxidative damage 55) The photograph demonstrates which of the following? A. Composite graft B. Full-thickness skin graft Correct Choice C. Mohs layer D. Split-thickness skin graft E. Dog ear A full-thickness skin graft (FTSG) is composed of the entire epidermis and dermis. Subcutaneous tissue must be removed since the fat may compromise the viability of the graft 56) Treatment of acne scarring would be most effective with which modality? A. Nd:YAG laserCorrect Choice B. Intense pulsed light C. Pulsed dye laser D. Photodynamic therapy E. Radiofrequency 15
  • 16. Treatment with the nonablative 1064-nm Q-switched Nd:YAG provides significant improvement in skin topography in patients with atrophic acne scars. The further improvements that are seen at 6- month follow-up suggest that dermal remodeling is a process that continues long after treatment 57) Which structual component of local anesthetics (e.g. lidocaine) is responsible for the onset of activity? A. Amine end B. None of the above C. Length of the carbon chain D. Aromatic ringCorrect Choice E. Intermediate chain Local anesthetics like lidocaine contain 3 principle structural elements. The aromatic ring determines the onset of activity, the intermediate chain defines the class (amide vs. ester), and the amine end is responsible for the duration of action 58) While removing sutures from an excision performed on the lateral neck, your patient reports decreased sensation of the helix and lobule of the ipsilateral ear. Which of the following nerves was likely injured during surgery? A. Spinal accessory nerve B. Greater auricular nerve Correct Choice C. Auricular branch of vagus nerve D. Lesser occipital nerve E. Auriculotemporal nerve Sensory innervation of the ear is provided by the greater auricular nerve, the auriculotemopral nerve, the lesser occipital nerve and the auricular branch of the vagus nerve. The greater auricular nerve is composed of the C2 and C3 branches of the cervical plexus and lies in the posterior triangle of the neck. It provides sensation to the helix, antihelix, antitragus, posterior auricle and the lobule. The spinal accessory nerve is a motor nerve which innervates the trapezius muscle. This nerve is also subject to injury in the lateral neck but would cause motor deficits rather than sensory deficits 59) Paradoxical darkening has been associated with Q-switched ruby, Q-switched alexandrite, Q- switched Nd:Yag treatment of which tattoo color? A. Green B. Black C. Blue D. Yellow E. RedCorrect Choice Paradoxic darkening of flesh-tone, red, and white tattoo inks with QSRL, Q-switched Nd:YAG, and Q-switched alexandrite lasers has been reported. The tattoo pigments most associated with this phenomenon are iron oxide and titanium dioxide. Ferric sulfide is converted to ferrous sulfate by the laser 60) Which tattoo pigment is most commonly associated with allergic reactions, eczematous and granulomatous? 16
  • 17. A. Cadmium sulfide B. Titanium dioxide C. Cobalt D. Mercuric sulfideCorrect Choice E. Carbon Allergic reactions have been reported with several different types of tattoo pigment. The most commonly associated tattoo pigment however is mercuric sulfide. Tattoo with mercuric sulfide produces a red color 61) Which dye is used to stain the Mohs section shown in the photograph? A. Eosin B. Toluidine blueCorrect Choice C. Cytokeratin 7 D. Hematoxylin E. Myeloperoxidase Toluidine blue is the stain in the photographed Mohs section. Since mast cells have been associated with tumor growth and are seen in increased numbers with basal cell carcinoma, staining of their metachromatic granules with toluidine blue can be an effective way of visualizing basal cell carcinoma on Mohs sections, particularly when the tumor is associated with fibrosis 62) The most common adverse reaction seen with betadine is: A. Ototoxity B. Seizures C. Corneal damage D. Allergic contact dermatitisCorrect Choice E. Teratogenicity Most common side effect seen with betadine is allergic contact dermatitis, secondary to the iodine componen 63) Which tattoo pigment has been most commonly associated with pseudolymphomatous reactions? A. Mercuric sulfideCorrect Choice B. Malachite C. Cadmium sulfide D. Phthalocyanine dyes E. Cobalt Some delayed type hypersensitivity reactions may lead to pseudolymphoma, most commonly caused by red tattoo pigment (mercuric sulfide 64) Which nerve provides the sensory innervation to the tragus? 17
  • 18. A. AuriculotemporalCorrect Choice B. Auricular branch of vagus C. Facial D. Glossopharyngeal E. Greater auricular The auriculotemporal nerve which is a branch of the mandibular branch of the temporal nerve innervates the tragus. The sensory innervation to the auricle is provided by the greater auricular nerve, and to a lesser extent the lesser occipital nerve. The external auditory canal, concha, and posauricular sulcus supplied variably by 3 nerves - the auricular branch of the vagus (CNX), the facial nerve (CN7), and the glossopharyngeal (CNIX 65) Treatment with which modality would be most effective for the patient shown? A. Pulsed dye laser B. Photodynamic therapy C. Radiofrequency D. Erbium:YAG laser E. Intense pulsed lightCorrect Choice The patient shown has multiple solar lentigines. Targeting the melanosomes in these benign, superficial pigmented lesions would be most successfully accomplished with the intense pulsed light source 66) The surgical instrument shown in the photograph is a: A. Stevens tenotomy scissor Correct Choice B. Metzenbaum scissor C. Castroviejo scissor D. Mayo scissor E. Iris scissor The instrument shown is a Stevens tenotomy scissor. This scissor is useful in dermatologic surgery for tissue cutting and blunt dissection 67) All of the following are alpha-hydroxy acids except: A. Tartaric acid B. Citric acid C. Glycolic acid D. Lactic acid E. Salicylic acidCorrect Choice Alpha-hydroxy acids are naturally occurring carboxylic acids found in many foods. The alpha- hydroxy acids include glycolic, lactic, malic, citric, and tartaric acids. Factors that determine the intensity of the peel include the concentration of the acid, pH, degree of buffering, vehicle formulation, frequency of application, conditions of delivery, and the length of time the acid is placed on the skin. Salicylic acid is a type of beta-hydroxy acid 18
  • 19. 68) Formation of granulomas is a potential complication of treatment with: A. Silicone Correct Choice B. Hydroxychloroquine C. Isotretinoin D. Autologous fat E. CO2 resurfacing Silicone is a synthetic, viscous compound which is composed of long polymers of dimethylsiloxanes. Silikon-1000 is one silicone product which is available in the United States and is FDA-approved for the ophthalmic treatment of complicated retinal detachment. The number 1000 refers to the product’s viscosity which is measured in centistokes. As a reference, water has a viscosity of 100 centistokes and mineral oil has a viscosity of about 350 centistokes. With respect to cosmetic applications, silicone is not FDA-approved for soft tissue augmentation. In addition to the occurrence of hypersensitivity reactions and product migration, there have been many reports of granuloma formation after silicone injection, even many years post-treatment 69) Which ocular structure is at most risk of injury from an erbium:YAG laser? A. Pupil B. Iris C. Retina D. Lens E. Cornea Correct Choice The erbium:YAG laser is an ablative laser whose target chromophore is water. The cornea is an aqueous structure and is therefore the most susceptible to damage from this laser 70) Activation of the procerus muscle causes A. Accentuation of the melolabial folds B. Perioral wrinkles C. Periocular wrinkles D. Wrinkles at the nasal rootCorrect Choice E. Wrinkles on the forehead The procerus muscle causes wrinkling at the nasal root and is often targeted with Botox therapy for improved cosmesis 71) The cutaneous lip and chin are divided into cosmetic units by which anatomic boundary? A. Vermillion border B. Nasolabial fold C. Mental creaseCorrect Choice D. Philtral ridge E. Marionette lines The term cosmetic unit is used to define regions of the face by specific characteristics such as color, texture, presence of hair, and prominence of sebaceous glands. In dermatologic surgery, it is 19
  • 20. preferable to conceal incisions within the boundary lines of cosmetic units. The mental crease, as demonstrated in the photograph, divides the cutaneous lip and chin 72) Treatment with which of the following lasers has been effective in psoriasis? A. Pulsed dye laser B. Alexandrite laser C. Ruby Laser D. Carbon dioxide laser E. Excimer laswerCorrect Choice The excimer laser is a 308 nm wavelength laser that has been used to treat psoriasis. The excimer laser has some advantages over ultraviolet therapy. By treating only involved skin, higher doses can be used and clearance may occur with fewer treatments 73) Which laser would not be a good choice for the treatment of a red cosmetic tattoo around the lips? A. Carbon dioxide B. Pulsed dye C. Nd:YAG D. Q-switched rubyCorrect Choice E. Alexandrite Q-switched lasers can be effectively used to remove tattoos. However, immediate darkening of white, flesh colored, and pink/red tattoos has been reported after treatment with Q-switched lasers. Both ferric oxide and titanium dioxide (found in tattoo pigments) may be both be reduced after heating by the laser and produce dramatic darkening 74) The anatomic structure identified in the photograph is called the: A. Antihelix B. Scaphoid fossa C. Concha D. Triangular fossaCorrect Choice E. Tragus The triangular fossa of the ear is shown in the photograph. This anatomic structure is bordered by the crura of the antihelix 75) Which of the following cosmetic injectables can be seen on routine dental x-rays? A. Botox B. Radiance Correct Choice C. Restylane D. Zyplast E. Cosmoderm 20
  • 21. Radiance is an injectable, biodegradable filler which is composed of calcium hydroxylapatite microspheres. Calcium hydroxylapatite is a normal constituent of bone and thus can be seen on radiographic imaging. Botox is botulinum toxin; Zyplast is bovine collagen; Cosmoderm is a non- animal form of collagen; and Restylane is a non-animal form of hyaluronic acid 76) The type of repair depicted in the photograph is a: A. Full-thickness skin graft B. Nasolabial flap C. Rotation flap D. Rhombic flap E. Island pedicle flap Correct Choice The island pedicle flap is a type of advancement flap commonly used for medial cheek defects. The closure can often be camouflaged in the melolabial fold 77) The use of dermabrasion to improve the cosmesis of a scar is best performed how long after the initial surgery? A. 1 year B. 1 week C. 6 months D. 6 weeksCorrect Choice E. 3 weeks Dermabrasion is the process of surgically planing or abrading the epidermis and dermis and is usually carried out with a rapidly rotating wire brush or diamond fraise. Following dermabrasion, reepithelialization from adnexal structures occurs with remodeling and replacement of collagen bundles in the papillary and reticular dermis. This can result in improvement of surgical or acne scars. When used in the post-operative period for surgical scars, dermabrasion is usually carried out 6 to 8 weeks following the procedure. Regional dermabrasions are routinely carried out under local anesthesia. Post-procedure complications include scarring, pigmentary alterations, persistent erythema, and infection. Contraindications for dermabrasion include recent use of isotretinoin and recent facelift, browlift or other procedure involving extensive undermining. Reactivation of herpes labialis can be prevented in the lip area by administering antiviral prophylaxis 78) Which cosmetic injectable provides dermal augmentation through the harvesting of a patient’s fibroblasts? A. Juvaderm B. Dermalogen C. Hylaform D. IsolagenCorrect Choice E. Restylane Isolagen is an emerging technology whereby a patient’s own fibroblasts are isolated from a skin biopsy, reproduced and then re-introduced into the patient’s treatment site 79) The tensile strength of a wound 6 months after surgery compared to intact skin is: 21
  • 22. A. 70% Correct Choice B. 15% C. 30% D. 50% E. 100% The tensile strength of a wound approaches 70% of normal skin strength at 8 weeks postoperatively. Wound healing is a process which takes many months, yet the tensile strength of a wound never exceeds 80% of the tensile strength of intact skin. It is therefore critical to use long- lasting subcuticular sutures to minimize the tension on a healing wound and prevent scar widening 80) Epinephrine should be avoided with anesthetics in which of the following conditions? A. Renal failure B. Hypothyroidism C. Liver failure D. PheochromocytomaCorrect Choice E. Diabetes Mellitus Epinephrine should be avoided in patients with a pheochromocytoma 81) Which ocular structure is at most risk of injury from an erbium: YAG laser? A. CorneaCorrect Choice B. Iris C. Retina D. Vitreous humor E. Lens Injury from lasers may occur via direct or indirect ocular exposure. Damage is generally wavelength specific. Laser that target hemoglobin or pigment may cause damage to the retinal pigment or vasculature while lasers that target water as a chromophore (carbon dioxide and erbium) can damage the cornea 82) Which one of the following lasers would be effective in the treatment of rhinophyma? A. Carbon dioxide laserCorrect Choice B. Alexandrite laser C. KTP laser D. Pulsed dye laser E. Ruby laser The carbon dioxide laser is a 10,600 nm laser that can be used to treat rhinophyma. The advantage of the carbon dioxide laser over steel or dermabrasion is that the laser is relatively bloodless. The erbium YAG can also be used to treat rhinophyma 83) A patient has a 1.5cm surgical defect on the right nasal sidewall that reveals the lateral cartilage. Which management option would be the least appropriate? 22
  • 23. A. Cheek transposition flap B. Primary closure C. Full-thickness skin graft Correct Choice D. Forehead flap E. Secondary intention Full-thickness skin grafts (FTSGs) depend upon a viable, vascular bed at the recipient site in order to survive. Exposed cartilage is an avascular tissue and therefore would not be able to support the metabolic requirements of a FTSG 84) Aging skin demonstrates all of the following characteristic changes except: A. Decreased number of Langerhans cells B. Increase in subcutaneous fat Correct Choice C. Hypertrophy of sebaceous glands D. Decreased dermal collagen E. Loss of elasticity The loss and redistribution of subcutaneous fat is a characteristic finding of the aging face. The forehead, temporal fossae, malar cheeks and perioral region are the most commonly affected areas. Knowledge of senescent changes in fat distribution has altered the cosmetic surgeon’s approach of the aging face to one which includes augmentation rather than one of simply lifting and tightening the skin 85) The classic purpose for serrated scissors is: A. For work on delicate areas around the eyes and the ears B. For tissue undermining C. For work on thicker tissues (back) D. For sharp dissections of tissue E. For gripping tissue while cuttingCorrect Choice Serrated scissors are used primarily for gripping tissue to prevent sliding while cutting 86) Which of the following lasers may cause milia formation as a post-procedure complication? A. Pulsed Dye B. ErbiumCorrect Choice C. ND-Yag D. KTP E. Excimer The erbium (2940 nm) and carbon dioxide (10,600 nm) lasers may both cause milia formation after laser skin resurfacing 87) Which of the following surgical prepatory solutions is teratogenic? A. Povidine-iodine 23
  • 24. B. Ethyl and Isopropyl alcohol C. Benzalkonium chloride D. Chlorhexidine E. HexachloropheneCorrect Choice Hexachlorophene or pHisoHex has shown to be toxic to developing embryos in animal studies and thus is prohibited for use in pregnant females. Chlorhexidine is a common ingredient in oral rinses(Peridex) and surgical prep solutions(Hibiclens) can cause ototoxicity if in contact with the middle ear and ocular toxicity if in contact with the eyes. Ethyl alcohol is effective against both gram positive and gram negative organisms, but only when the skin is dry. Both Povidine-Iodine (Betadine)and Benzalkonium chloride (Zephiran) can cause an allergic contact dermatitis 88) What is the most appropriate management of the following surgical complication? A. Antibiotic therapy B. Observation Correct Choice C. Incision and drainage D. Debridement E. Pulse dye laser If the epidermal surface of a graft becomes black and necrotic, it does not necessarily signify graft failure. The epidermal portion of the graft may slough with subsequent re-epithelialization. In this situation, the best treatment is observation 89) A physician using a carbon dioxide laser to treat verruca should be aware that all of the following complications can occur except: A. Recurrence of lesion B. Ocular damage C. Transmission of viral disease D. PurpuraCorrect Choice E. Scarring The carbon dioxide laser uses a 10,600 nm wavelength to target water as a chromophore. Lasers that target water may damage the cornea. The carbon dioxide laser can be used to destroy epidermal and dermal lesions such as warts. HPV virus has been recovered in the laser plume after treatment with carbon dioxide laser. Bovine papilloma virus recovered in the plume was even found to transmit disease to calf skin. Scarring and recurrence have also been found to be potential complications 90) A Z-plasty is performed to: A. Correct dog-ears B. Obtain wound eversion C. Shorten the length of an excision D. Decrease tension on a wound Correct Choice E. Make use of excess tissue 24
  • 25. A Z-plasty is a transposition flap used most commonly in the treatment of contractures and scars. The main advantages to performing a Z-plasty are to decrease the tension on a wound, change the orientation of a scar, and camouflage a scar by breaking it up into smaller components 91) Which of the following cosmetic injectables is FDA-approved for the treatment of lipoatrophy? A. Botox B. Radiance C. Silicone D. SculptraCorrect Choice E. Isolagen Sculptra (called New-Fill outside of the US) is a biodegradable filler composed of poly-L-lactic acid, the same material used in absorbable sutures. It is biocompatible and nonallergenic and was approved by the FDA in August 2004 for the treatment of HIV-associated lipoatrophy 92) A history of anaphylaxis is a contraindication to treatment with which cosmetic injectable? A. Cosmoderm B. Radiance C. Sculptra D. RestylaneCorrect Choice E. Myobloc Restylane is a hyaluronic acid gel produced by the Streptococcus species of bacteria. The package insert for Restylane warns that its use is contraindicated in patients with “severe allergies manifested by a history of anaphylaxis or the presence of multiple severe allergies 93) What part of the eye may be damaged by exposure to irradiation from the carbon dioxide laser? A. Sclera B. Lens C. Retina D. CorneaCorrect Choice E. Iris The carbon dioxide laser operates at a wavelength of 10,600 nm and targets water as a chromophore. Because of the high water content of the cornea, it may be damaged by exposure to irradiation from the carbon dioxide laser. Exposure to the erbium may cause corneal damage as well 94) All of the following statements are true regarding nerves blocks on the face except: A. Anesthesia is placed around a nerve trunk B. Lidocaine with epinephrine may be used C. Numbness occurs in areas other than the operative site D. Hemostasis is achieved using epinephrineCorrect Choice E. Tissue distortion is minimized 25
  • 26. While regional nerve blocks offer many advantages in cutaneous surgery, hemostasis is not one of them. Since the anesthetic is injected distant to the operative site, the vasoconstrictive effects of the epinephrine, and hence hemostasis, are not provided at the surgical site 95) The post-operative complication shown in the photograph is most commonly seen with which type of reconstruction? A. Full-thickness skin graft B. Rhombic flap C. Nasolabial flap Correct Choice D. Secondary intention E. Rotation flap Trap door deformity is believed to result from insufficient undermining. This surgical complication is most often associated with the nasolabial transposition flap. Intralesional corticosteroids may be beneficial in improving the cosmetic outcome 96) The pain associated with Botulinum A Exotoxin injection is attributed to the: A. Needle gauge B. Preservative-free salineCorrect Choice C. Temperature D. PH E. Exotoxin Botulinum A exotoxin is used for multiple reasons, most often for the treatment of dynamic facial lines. In a double-blind, randomized controlled study, investigators found that botulinum A exotoxin reconstituted with preservative-containing saline less painful than with preservative-free saline 97) An M-plasty is performed to: A. Make use of excess tissue B. Reorient a scar C. Decrease tension on a wound D. Obtain wound eversion E. Shorten the length of an excision Correct Choice An M-plasty is a variation of the fusiform excision where either one or both ends of the ellipse are modified. The main advantages to performing an M-plasty are to shorten the length of an excision and to correct dog-ears 98) Botulinum toxin A (Botox) is FDA-approved for the treatment of: A. Platysmal bands B. Migraine headaches C. HyperhidrosisCorrect Choice D. Masseter hypertrophy 26
  • 27. E. Crow’s feet Botox has been recently FDA-approved for the treatment of hyperhidrosis. A CPT procedure code has been assigned to treatment with Botox for this indication with variable reimbursement rates by insurance companies 99) The antiptosis subdermal suspension threads used in facial rejuvenation are composed of: A. Polyglycolic acid B. Poly-L-lactic acid C. Polydioxanone D. Polyglactin E. PolypropyleneCorrect Choice The Aptos subdermal suspension thread technique is a minimally invasive procedure which targets the ptotic changes seen with facial aging. 2-0 and 3-0 polypropylene threads are tunneled in the dermis to lift and suspend the skin and subcutaneous tissue 100) Which one of the following best estimates the percent strength of a wound one month after surgery with a primary closure? A. 90% B. 70% C. 80% D. 30% E. 50%Correct Choice One month after surgery with a primary closure a wound is approximately at 50% strength. Two weeks after surgery it is at 5% strength. Three weeks after surgery it is at 20%. It never reaches 100% of its original strength, its maximum strength is 90% 101) Topical vitamin K has been shown to: A. Increases epidermal differentiation B. Improve skin hydration C. Decrease epidermal pigmentation D. Reduce severity of postoperative purpuraCorrect Choice E. Improve fine winkling Topical vitamin K reduced post later treatment purpura when applied after (not before) treatment of telangectasia with the 585 nm pulsed dye laser 102) Which type of collagen is the first to be deposited in a healing wound? A. Type I collagen B. Type II collagen C. Type III collagenCorrect Choice D. Type IV collagen 27
  • 28. E. Type VII collagen Coagulation/Inflammation, tissue formation, and scar/remodeling are the three phases important in wound healing. Collagen deposition occurs during the second phase. Fibroblasts migrate into the wound along the fibrin-fibronectin matrix deposited in the initial clot. The fibroblasts subsequently produce type I and III collagen, elastin, and proteoglycans. Collagen type III is the predominant collagen in early wound healing 103) All of the following statements are true regarding imiquimod (Aldara) except: A. It is FDA-approved for the treatment of actinic keratoses B. It is FDA-approved for the treatment of a 1.9 cm superficial basal cell on the chest C. It is FDA-approved for the treatment of a 2.0 cm superficial basal cell on the back D. It is FDA-approved for the treatment of a 1.5 cm superficial basal cell on the leg E. It is FDA-approved for the treatment of a 1.2 cm superficial basal cell on the scalpCorrect Choice Imiquimod (Aldara) is FDA-approved for the treatment of actinic keratoses and the treatment of superficial basal carcinoma. With respect to basal cell carcinoma, treatment is indicated for primary tumors that are 2.0 cm or less, and that are located on areas of the body excluding the face, scalp and anogenital region 104) A patient has a 1.8 cm nodular basal cell carcinoma on his neck. Which of the following criteria would be an indication for Mohs micrographic surgery? A. Anatomic location B. Size of tumor > 1.5 cm C. Tumor is contiguous D. Previous radiationCorrect Choice E. Histologic subtype Previous irradiation is an indication for Mohs surgery because the effects of radiation are similar histologically and biologically to scarring. Specifically, the tissue can show fibrosis and alter the tumor’s growth pattern. In terms of location, the neck is not a high risk site. With respect to pathology, nodular basal cell is not an aggressive subtype of basal cell carcinoma. Size of tumor > 2 cm would be an indication for Mohs. Finally, that the tumor grows contiguously is not an indication but an absolute requirement for Mohs surgery 105) Which porphyrin is targeted in aminolevulinic acid-photodynamic therapy? A. Uroporphyrinogen B. Coproporphyrinogen C. Porphobilinogen D. Protoporphyrin IXCorrect Choice E. Coproporphyrin III Protoporphyrin IX is the substrate for the final rate-limiting step of heme synthesis. This enzymatic reaction is catalyzed by ferrochelatase. The preferential uptake of aminolevulinic acid by cancerous and pre-cancerous cells results in higher concentrations of protoporphyrin IX. This can then be selectively targeted by photodynamic therapy 106) Which of the following sun protection agents is currently seeking FDA approval? 28
  • 29. A. MexorylCorrect Choice B. Titanium dioxide C. Avobenzone D. Zinc oxide E. Octocrylene Mexoryl is considered the best UVA sunscreen currently available. Its broad-spectrum characteristics allow sunscreens to be manufactured with very high SPF factors. Various formulations of mexoryl have long-been available in Europe, Canada, Australia, Asia, Mexico and South America 107) Which of the following local anesthetic agents should not be used in children? A. Benzocaine B. PrilocaineCorrect Choice C. Etidocaine D. Mepivicaine E. Bupivicaine Prilocaine should bot be used in children given the risk of methemoglobinemia.Methemoglobulinemia in children can occur from exposure to oxidizing substances such as aniline dyes, prilocaine or pyrimidine. In methemoglobinemia, the iron in hemoglobin is oxidized from the ferrous state (Fe 2+) to ferric (Fe3+) resulting in the inability to transport oxygen and carbon dioxide. Clinically, this condition results in cyanosis 108) When can a patient who undergoes follicular-unit hair transplantation expect hair growth of the grafted follicles to begin? A. 2 to 3 months Correct Choice B. 7 to 10 days C. 6 months D. 9 months E. 1 month The use of follicular-unit grafts, which contain one to four hair follicles, represents the advancement in both surgical technique and aesthetic outcome in the field of hair transplantation. The grafted hair follicles typically begin to grow within 8 to 10 weeks of implantation and are expected to survive for the individual’s lifetime 109) Which of the following is a potential side effect of ambulatory tumescent liposuction? A. Pulmonary embolus B. Abdominal perforation C. Decreased appetite D. Muscle atrophy E. Breast enlargement Correct Choice Breast enlargement is a relatively common and unexpected side effect of tumescent liposuction. Since the majority of these patients report increased breast size in the absence of weight gain, 29
  • 30. some authors postulate shifting hormone ratios as the etiology of this paradoxical breast augmentation. Abdominal perforation, respiratory failure and pulmonary embolus are complications that are seen almost exclusively in liposuction patients that receive general anesthesia or intravenous sedation 110) A patient with a squamous cell carcinoma of the helical rim had excisional surgery followed by closure of the wound with an advancement flap. Which would be the most appropriate antibiotic for this patient to receive postoperatively? A. Trimethoprim-sulfamethoxazole B. Cephalexin C. Ciprofloxacin Correct Choice D. Erythromycin E. Dicloxacillin Infection after cutaneous surgery is typically due to Staphylococcal and Streptococcal organisms. However, Pseudomonas aeruginosa may be normal flora of the ear and thus can potentially complicate ear surgery by causing severe infection of the external ear canal (malignant otitis externa). Ciprofloxacin would be the antibiotic of choice for this patient because it is effective against Pseudomonas species 111) The repair demonstrated in the photograph is a: A. Advancement flap Correct Choice B. Rotation flap C. Full-thickness skin graft D. Split-thickness skin graft E. Transposition flap The postauricular flap is useful for repairing extensive defects of the helical rim, particularly when cartilage is involved. It is an example of an advancement flap 112) The use of EMLA cream is contraindicated in patients with which of the following? A. Sickle cell anemia B. Peripheral neuropathy C. MethemoglobinemiaCorrect Choice D. Atopic dermatitis E. Deomycin allergy The most serious adverse effect of eutectic lidocaine and prilocaine (EMLA) is methemaglobinemia. A metabolite of prilocaine oxidizes hemoglobin to methemoglobin, which is less efficient in release of oxygen leading to tissue hypoxia. Patients with congenital or idiopathic methemoglobinemia or infants under 1 year of age are at higher risk for the development of this side effect 113) The type of repair depicted in the photograph is a: A. Rotation flap B. Island pedicle flap 30
  • 31. C. Full-thickness skin graft D. Rhombic flap Correct Choice E. Nasolabial flap The rhombic flap is a type of transposition flap. The classic rhombic flap is designed with two 60 degree angles and two 120 degree angles. The point of maximum tension is at the closure of the donor site 114) In organ transplant recipients, all of the following factors increase susceptibility to the development of skin cancer except: A. CD8 lymphocytopeniaCorrect Choice B. Fair skin (Fitzpatrick types I-III) C. Duration of immunosuppression D. History of chronic sun exposure E. History of HPV infection All organ transplant recipients are at increased risk for the development of cutaneous malignancies. However, the above-mentioned factors, with the exception of choice E, place these individuals at further risk. CD4 lymphocytopenia, rather than CD8, is another identified risk factor as is older age, history of actinic keratoses and history of skin cancer 115) The anatomic structure identified in the photograph is called the: A. Philtral ridgeCorrect Choice B. Cupid’s bow C. Vermillion border D. Soft triangle E. Columella The philtral ridges are located at the upper middle lip and form part of the aesthetically important cupid’s bow 116) What is the tattoo pigment responsible for most lichenoid reactions? A. Titanium dioxide B. Mercuric sulfideCorrect Choice C. Carbon D. Chromates E. Iron oxide Lichenoid tattoo reactions are not as common as eczematous reactions, both of which are most commonly caused by mercuric sulfide. Lichenoid reaction are likely to be mediated by delayed hypersensitivity to a lymphocytic T-cell infiltrate 117) Which part of the eye is most likely to be damaged to exposure to a pulsed dye laser? A. Sclera 31
  • 32. B. Cornea C. RetinaCorrect Choice D. Lens E. Conjuctiva The pulsed dye laser operates at 585-nm and targets hemoglobin as a chromophore. It can pass through the cornea and damage choroidal and retinal vasculature. Several infrared pigment-specific lasers (eg, ruby, alexandrite, Nd:YAG) may also damage the retina by targeting the retinal pigment 118) All of the following are true statements regarding the immunohistochemical stain Melan-A except: A. Reliably stains desmoplastic melanoma Correct Choice B. Sensitive marker for melanocytic nevi C. Antigen present in 80-100% of melanomas D. Effective in frozen and paraffin-embedded tissue E. Recognized by CD8+ T cells E Melan-A is a 22-kDa cytoplasmic melanosome-associated glycoprotein also known as MART-1 (melanoma antigen recognized by T cells). It is a sensitive marker for both melanocytic nevi and melanoma and may be used on frozen tissue sections as well as paraffin-embedded tissue. Although the antigen is present in > 80% of melanomas, it does not reliably stain desmoplastic or spindle cell melanomas 119) A patient with a basal cell carcinoma on the lower eyelid has Mohs surgery. Once tumor-free margins were obtained, the surgeon repairs the wound with a full thickness skin graft. Which of the following statements is true about this repair method? A. Healing by secondary intention would minimize the risk of ectropion B. Graft should be at least 25% smaller than the defect C. Graft should be the same size as the defect D. Graft rarely survives in this location E. Graft should be at least 25% larger than the defectCorrect Choice To account for wound contraction and minimize the risk of ectropion, full thickness skin grafts on the lower eyelid should be sized such that the graft is at least 25% larger than the actual size of the wound. Jewett BS and Shockley WW. Reconstructive options for periocular defects. Otolaryngol Clin North Am 2001; 34(3):601-625 120) Which tattoo pigment has most commonly been associated with phototoxic reactions? A. Titanium dioxide B. Mercuric sulfide C. Cadmium sulfideCorrect Choice D. Carbon E. Iron oxide 32
  • 33. Cadmium sulfide produces a yellow tattoo. Phototoxic reactions are most commonly caused by cadmium sulfide tattoo pigment. Red tattoos have also been associated with phototoxic tattoo reactions. However, this is thought thought to be due to the addition of cadmium sulfide which enhances and brightens the red color 121) Which of the following lasers has the greatest depth of penetration in the skin? A. Diode (800 nm) B. CO2 (10,600 nm) C. Erbium: YAG (2940 nm) D. Pulsed dye laser (585 nm) E. Nd: YAG (1064 nm) Correct Choice The Nd: YAG laser emits energy at 1064nm near the infrared range and penetrates the skin to the level of the deep dermal blood vessels. The target chromophore for this laser is melanin, thus its primary use is in treating lesions such as nevus of Ota and removing black tattoo pigment. Although both the erbium:YAG and CO2 lasers have longer wavelengths than the Nd: YAG, they are ablative lasers which only penetrate to the level of the stratum corneum and superficial epidermis, respectively 122) Which of the following suture materials is most likely to cause infection? A. PDS (polydioxanone) B. Silk Correct Choice C. Prolene (polypropylene) D. Nylon E. Vicryl (polyglactin) Silk is a nonabsorbable braided suture which has been shown to aid in the production of infection. It is believed that the interstices of braided or twisted suture materials enhance the potential for developing infection by providing privileged sites which harbor bacteria 123) A patient with a large nasal tip defect had reconstruction of his wound with a forehead flap. After what postoperative interval should the patient schedule the inset of this flap? A. 2 months B. 6 months C. 3 weeks Correct Choice D. 3 months E. 1 week The forehead flap is often the reconstructive option of choice for large nasal defects. The flap has a vertically-oriented blood supply which is supplied primarily from the supratrochlear arteries. Pedicle division and inset of this flap are typically performed 3 weeks after the initial surgery 124) Which of the following criteria carries the worst prognosis for a patient with a squamous cell carcinoma? A. Size of tumor > 1 cm 33
  • 34. B. Perineural invasion C. Depth of invasion > 4 mm Correct Choice D. Immunosuppression E. Anatomic location Many factors contribute to the aggressive behavior of cutaneous squamous cell carcinoma. For example, tumors in immunosuppressed patients and tumors with the histologic subtypes desmoplastic and acantholytic are often biologically aggressive squamous cell carcinomas. Compared with other risk factors, depth of invasion >4 mm and size > 2cm demonstrate the greatest risk for metastasis. 125) The relaxed skin tension lines overlying the scapula are: A. Oriented horizontally B. Parallel to the underlying muscle groups C. Oriented verticallyCorrect Choice D. Do not exist over the scapula E. Oriented circumferentially Relaxed skin tension lines (RSTLs) are creases in the skin that are present at rest and develop as a consequence of the decreased elasticity that occurs with aging and solar damage. These lines lie perpendicular to the underlying musculature. They tend to run vertically on the upper back whereas on the central trunk, they are typically circumferential. Knowledge of RSTLs is important in cutaneous surgery because placing incisions within these lines will ensure the most favorable cosmetic result 126) Which of the following cosmetic injectables binds water to create volume, has a low allergic reaction profile, and lasts 6 to 12 months? A. Silicone B. Calcium hydroxylapatite C. Human-derived collagen D. Botulinum toxin E. Hyaluronic acid Correct Choice Hyaluronic acid is a natural component of human connective tissue. As a cosmetic filler, hyaluronic acid is an attractive alternative to currently available filler substances: its ability to bind water creates volume and plumps the skin; its duration of action of approximately 6-12 months is appreciably longer than conventional collagen; and since hyaluronic acid is chemically identical across all species, hypersensitivity reactions are rare 127) Which of the forms of electrosurgery can work in a wet field? A. ElectrocauteryCorrect Choice B. Electrodessication C. Electrocoagulation D. Electrosection E. Electrofulguration Electrocautery is the only one to work in a wet field as it has no current and works via heat 34
  • 35. 128) Which of the following complications has been reported with infraorbital injections of Botox? A. FestooningCorrect Choice B. Photophobia C. Blindness D. Astigmatism E. Nystagmus Festooning of the lower eyelid is a recently reported complication of infraorbital Botox injections. The affected patient had a prior lower lid blepharoplasty which likely weakened the orbicularis oculi muscle. The author postulates that festooning resulted from further muscle weakness due to Botox treatment 129) When is it appropriate to use this surgical technique? A. To shorten the length of the scar B. To change direction of a scar that is deformingCorrect Choice C. When there is insufficient tissue for rotation flap D. To prevent bleeding E. When it is desirable not to extend an ellipse Z-plasty is used to change the direction of scar that is deforming or to elongate a scar that is contricting. Common sites are medial canthus, side of the face and forehead 130) Which of the following sutures is the first to be absorbed? A. Polyglycolic acid B. Polydioxanone C. CatgutCorrect Choice D. Polypropylene E. Polyglactin 910 Catgut has a variable rate of absorption but typically lasts about 7-14 days. Thus, it would be the first suture to be absorbed. Polyglactin 910 (Vicryl) and polyglycolic acid (Dexon) are both absorbed in about 90 days. Polydioxanone (PDS) is an absorbable monofilament which lasts approximately 180 days. Polypropylene (Prolene) is not an absorbable suture. 131) The most appropriate suture for closing a wound on the buccal mucosa is: A. Braided nylon B. SilkCorrect Choice C. Prolene D. PDS E. Monocryl 35
  • 36. The most appropriate suture for closing a wound on the buccal mucosa is silk. Other sutures may be used as well, but classically, silk is used because of its soft nature 132) The most appropriate laser to treat a port wine stain on an infant’s cheek would have which of the following wavelengths? A. 488 nm B. 585 nm Correct Choice C. 532 nm D. 810 nm E. 694 nm The 585nm pulsed dye laser (PDL) targets intravascular oxyhemoglobin and is considered the treatment of choice for most benign vascular lesions. The original PDL had a wavelength of 577 nm which was later modified to 585 nm to achieve deeper penetration yet still maintainvascular specificity 133) Which of the following absorbable sutures lasts the longest? A. Polydioxanone Correct Choice B. Catgut C. Polypropylene D. Polyglycolic acid E. Polyglactin 910 Polydioxanone (PDS) is an absorbable monofilament which lasts approximately 180 days. Catgut has a variable rate of absorption but typically lasts about 7-14 days. Polyglactin 910 (Vicryl) and polyglycolic acid (Dexon) are both absorbed in about 90 days. Polypropylene (Prolene) is not an absorbable suture 134) The most important function of a postoperative wound dressing is: A. Provide a barrier to infection B. Decrease tension on the wound C. Create a moist environment D. Prevent suture dehiscence E. Immobilize the wound Correct Choice Providing hemostasis to a wound is the most important function of a wound dressing. This is particularly critical in the first few hours postoperatively when the vasoconstrictive effects of epinephrine are wearing off and the wound is at risk of bleeding or hematoma formation. The provision of a barrier to infection and creation of a moist, occlusive environment are also necessary functions of a dressing but are not as critical as immobilizing the wound and ensuring hemostasis 135) While contemplating the repair of a large cheek defect after Mohs surgery, you begin to anesthetize an elderly lady who weighs 110 lbs. What is the maximum amount of 1% lidocaine with 2.5% epinephrine that this patient can receive? A. 500 ml 36
  • 37. B. 350 ml C. 35 ml Correct Choice D. 50 ml E. 10 ml he maximum recommended dosage of 1% lidocaine with 2.5% epinephrine in adults is 7mg/kg. The patient weighs 110 lbs (or 50 kg) which allows her 350 mg or 35 ml (1% lidocaine has 10mg of lidocaine perml) of the anesthetic 136) Which potential complications is seen exclusively with phenol peels? A. Laryngeal edema Correct Choice B. Prolonged erythema C. Milia D. Toxic shock syndrome E. Hypopigmentation The principle concept of chemical peeling is to utilize a chemical agent to produce a controlled wound in the skin. Many different types of side effects can occur with chemical resurfacing and are typically related to the depth of the wound. Complications which are common to all peeling agents include: milia; acne; pigmentary changes; contact dermatitis; scarring; infection; prolonged erythema; textural changes and cold sensitivity. Unlike other peeling agents, phenol has the potential to cause severe adverse reactions which are exclusive to its use only. They include atrophy, cardiac arrhythmias and laryngeal edema 137) Reticulate eythema is a side effect seen with which treatment? A. Cryotherapy B. Pulsed dye laser C. Diode laserCorrect Choice D. Mesotherapy E. Sclerotherapy Reticulate erythema is a recently-reported side effect of diode laser treatment. High energy fluences and a history of chilblains are believed to be predisposing risk factors. Laser treatment should be discontinued at the first sign of this complication 138) Alopecia neoplastica is most commonly associated with which malignancy? A. Lymphoma B. Squamous cell carcinoma C. Breast carcinoma Correct Choice D. Colon carcinoma E. Melanoma While various malignancies have a predilection for metastasis to the scalp where they can cause hair loss, alopecia neoplastica is most commonly associated with metastatic breast carcinoma. 139) The ‘snap test’ is performed prior to surgery at which anatomic location? 37
  • 38. A. Mouth B. Neck C. Ear D. Hand E. EyelidCorrect Choice The snap test determines the laxity of the lower eyelid. It should be performed prior to surgery, laser resurfacing or Botox injections of the lower eyelid to assess the potential for ectropion formation. Ratner D et al. Cutaneous laser resurfacing. J Am Acad Dermatol 1999; 41(3):365-389 140) Which anesthetic has the shortest duration of action? A. Tetracaine B. ProcaineCorrect Choice C. Mepivacaine D. Bupivacaine E. Prilocaine Procaine has the shortest duration of action, only lasting about 30-60 minutes. It is an ester anesthetic and is commonly used in dentistry 141) For patients with AJCC stage III and IV melanoma, which of the following adjuvant treatments has shown the greatest increase in survival? A. Melacine B. CanvaxinCorrect Choice C. Interferon alpha-2B D. Dacarbazine E. M-Vax The current paradigm for treatment of advanced-stage melanoma includes tumor reduction via surgery followed by adjuvant systemic therapy to manage residual occult disease. Although interferon alpha 2B and dacarbazine are FDA-approved adjuvant therapies, neither has significantly increased survival rates. Canvaxin is a polyvalent vaccine that contains more than 20 tumor and melanoma-associated antigens. Currently in phase 3 trials, postsurgical immunotherapy with Canvaxin has shown in retrospective matched-pair studies to prolong overall survival of stage III and IV melanoma patients by more than 175 percent 142) What is the ratio of sodium bicarbonate to 1% lidocaine in a buffered lidocaine solution? A. 1:25 B. 1:5 C. 1:10Correct Choice D. 1:1 E. 1:2 The use of sodium bicarbonate in 1% lidocaine reduces the acidity of the local anesthetic. Unbuffered lidocaine has pH of approximately 5.5 -7.0 and lidocaine with epinephrine has a pH of approximately 3.5 – 5.0. The addition of sodium bicarbonate at a ratio 1:10 raises the pH to a more physiologic level, thereby reducing the stinging sensation associated with injection. However, the addition of bicarbonate also reduces the shelf-life of the lidocaine 38
  • 39. 143) The use of imiquimod (Aldara) for the treatment of superficial basal cell carcinoma is advocated at which treatment regimen? A. Five times per week for 4 weeks B. Five times a week for 6 weeks Correct Choice C. Three times a week for 10 weeks D. Three times per week for 4 weeks E. Three times a week for 6 weeks Imiquimod (Aldara) was FDA-approved in 2004 for the treatment of superficial basal cell carcinoma. The recommended treatment schedule is once daily, five times per week for a total of six weeks. Geisse J et al. Imiquimod 5% cream for the treatment of superficial basal cell carcinoma: results from two phase III, randomized, vehicle-controlled studies 144) When would use of this technique be most beneficial? A. To shorten the length of the woundCorrect Choice B. To prevent post-operative bleeding C. To prevent pincushioning D. To prevent post-operative infection E. To increase the width of the wound M-plasty is used to shorten wound length and is often when it is not desirable to extend the ellipse into a nearby structure 145) Which anatomic structure is likely to be severed during this repair? A. Marginal mandibular nerve B. Labial artery Correct Choice C. Mental nerve D. Angular artery E. Buccal nerve The photograph demonstrates a wedge resection which is used to repair defects of the lower lip. During this repair, the labial artery will be severed since it courses superficially between the mucosa and the underlying muscle 146) The rhytids identified in the photograph are referred to as: A. Marionette lines B. Bunny linesCorrect Choice C. Nasolabial folds D. Crow’s feet E. Relaxed skin tension lines Bunny lines are located on the dorsum of the nose and are formed by the contraction of the nasalis muscle 147) Which of the following topical antibacterial agents may cause neutropenia? 39
  • 40. A. Silver sulfadiazine Correct Choice B. Mupirocin C. Bacitracin D. Polymyxin E. Povodine-iodine Silver sulfadiazine is most commonly used to prevent infection in second and third-degree burn patients. Rare cases of leukopenia, neutropenia and kernicterus have been reported in patients using this topical antibacterial agent. Consequently, silver sulfadiazine should be used cautiously and avoided in infants, nursing mothers and pregnant women. Silver sulfadiazine also has the potential to cause a hypersensitivity reaction in patients with sulfa allergy 148) Which material is used to coat the jaws of the needle holder in the photograph? A. Tungsten carbideCorrect Choice B. Copper C. Gold D. Nickel E. Stainless steel Tungsten carbide is believed to be a harder more durable material that allows for better grasping of the needle. Needle holders with tungsten carbide jaws usually have gold-plated handles 149) Jessner’s solution contains all of the following except: A. Ethanol B. Salicylic acid C. Resorcinol D. Lactic acid E. Glycolic acidCorrect Choice Jessner’s solution is a combination of resorcinol, salicylic acid, and lactic acid in ethanol that is used as a superficial peeling agent. The advantage of Jessner’s solution is that timing is unnecessary and neutralization is not performed. The lactic acid is an alpha hydroxy acid 150) The following are true statements regarding the design of an excisional ellipse except: A. Ellipse includes pathologic and normal tissue B. Incision lines are placed in relaxed skin tension lines C. Long axis of ellipse is oriented perpendicular to free margins D. Length of ellipse is at least 3 times its total width E. The angle at each of the apices is 15ºCorrect Choice Thoughtful design of an elliptical excision is important for obtaining optimum tissue diagnosis, tumor clearance, wound closure and cosmetic result. The above statements are all true except that the angles at the apices of the ellipse should be 30º 151) Blue nodules in the skin are a potential complication of which cosmetic injectable? 40
  • 41. A. Silicone B. Radiance (hydroxyapatite) C. Cosmoderm (collagen) D. Restylane (hyaluronic acid)Correct Choice E. Sculptra (poly L-lactic acid) Although hyaluronic acid is believed to be inert in humans, rare reports exist of granulomatous foreign body reactions to this material. Skin biopsy of a Restylane nodule demonstrates multinucleated giant cells surrounding a blue amorphous material 152) Which of the following is true regarding a Baker-Gordon peel? A. Croton oil is considered the most important component for efficacyCorrect Choice B. It contains resorcinol C. 88% phenol is the most important component for efficacy D. It contains salicytic acid E. It is considered a medium depth peel Croton oil is considered the most important component for efficacy. It is a epidermolytic that enhances penetration. A Baker Gordon peel is a deep peel that contains 88% phenol, tap water, croton oil, and septisol 153) The surgical instrument shown in the photograph is a: A. Towel clamp B. Iris scissor C. HemostatCorrect Choice D. Gradle scissor E. Needle holder A hemostat is shown in the photograph. This instrument is commonly used in cutaneous surgery for clamping off blood vessels 154) The use of EMLA cream is contraindicated in patients with which of the following? A. Neomycin allergy B. Methemoglobinemia Correct Choice C. Atopic dermatitis D. Peripheral neuropathy E. Sickle cell anemia EMLA is a topical anesthetic composed of a eutectic mixture of 2.5% lidocaine and 2.5% prilocaine. The major concern when using EMLA is the potential risk of methemoglobinemia. Patients with glucose-6-phosphate deficiency and patients taking methemoglobin-inducing agents (dapsone, phenytoin, sulfonamides) are more susceptible to developing methemoglobinemia. In addition, care should be taken when using EMLA in infants less than 3 months of age because of the incomplete maturation of their NADH-methemoglobinemia reductase system 155) Signs and symptoms of lidocaine toxicity include all of the following except: 41
  • 42. A. OtotoxicityCorrect Choice B. Circumoral numbness C. Seizure D. Slurred speech E. Nystagmus The first signs of lidocaine toxicity are CNS symtpoms that resemble inebriation with alcohol. These symptmoms include stupor, dysarthria, circumoral numbenss and dizziness. Further increases in toxicity leads to nausea, metallic taste, twitching, and seizures. Ototoxicity is not one of the toxic manifestations seen with lidocaine toxicity. Without epinephrine the maximum dose of lidocaine 4mg/kg. For a 70 kg individual, this is 300 mg or 30 ml of a 1% lidocaine solution. For a preparation of lidocaine without epinephrine the maximum dose is 7mg/kg. For a 70 kg individual, this is 500 mg or 50 ml of a 1% solution 156) To avoid injury to the facial nerve, undermining in the region of the temple should be performed at which anatomic level? A. Reticular dermis B. Subcutaneous fat Correct Choice C. Above the frontalis muscle D. Above superficial fascia (SMAS) E. Dermal-epidermal junction In the region of the temple, lateral to the eyebrow, the temporal branch of the facial nerve lies in the superficial fascia. Once above the eyebrow, the nerve is located deep to the frontalis muscle. In the region of the temple, it is therefore prudent to undermine in the superficial subcutaneous fat 157) Which of the following histochemical stains can be used intraoperatively during Mohs micrographic surgery to confirm the diagnosis of extramammary Paget's disease? A. S-100 B. Myeloperoxidase C. Toluidine blue D. Neuron-specific enolase E. Cytokeratin 7 Correct Choice Cytokeratin 7 is a structural component of the cytoskeleton that stains poorly differentiated tumors of the epithelium. This low molecular weight cytokeratin positively stains Paget cells. Neuron- specific enolase stains merkel cell carcinoma, toluidine blue stains mast cells and myeloperoxidase stains cells with myeloid differentiation such as leukemia cells. S-100 protein is a non-specific stain that is commonly used as an adjunctive marker in the diagnosis of melanoma 42